MARK final

¡Supera tus tareas y exámenes ahora con Quizwiz!

50. A __________ gap reflects the difference between customers' expectations and the firm's perception of those customer expectations. A. quality B. knowledge C. standards D. delivery E. communication

50. B - This is the definition of knowledge gap

46. Once a marketing researcher is ready to move beyond preliminary insights to specific, informed questions, the researcher is ready to conduct A. quantitative research. B. data warehousing. C. syndicated marketing surveys. D. qualitative research. E. research design.

46. A - Qualitative research is often done in part to obtain preliminary insights. Once this is complete and the researcher understands the research topic, the project can proceed to conduct quantitative research, attempting to obtain definitive answers to the research questions.

48. After a firm has identified the various stakeholders and their issues and gathered the available data, all parties relevant to the decision should engage in brainstorming and evaluating alternatives. __________ then review and refine these alternatives, and choose a course of action. A. Managers B.The firm's lawyers C.Key customers D.Community leaders E.All stakeholders

48. A - Although all stakeholders should be involved in the earlier steps of the process, management is responsible for choosing the final course of action.

59. In a competitive market, perceived value is determined by consumers mostly A. by quantitative analysis of brand personalities. B. in relationship to the value of competitors' offerings. C. by weighing primary versus secondary benefits. D. by trying out different products. E. through brand association and brand licensing.

59. B - The perceived value of a good or service is the relationship between its benefits and its costs. In a competitive market, these benefits and costs will be easiest to evaluate in comparison to competitive offerings

59. At the BMW plant in Spartanburg, South Carolina, rather than use a typical order-to-delivery process, it has suppliers deliver parts every four hours when the plant is in operation. BMW uses a(n) ______ inventory control system A. cross-docking B. lead time C. just-in-time (JIT) D. pick ticket E. UPC

59. C - Just-in-time (JIT) and quick response (QR) both refer to inventory management systems designed to ship smaller quantities more frequently

59. Which of the following is least likely to be a source of ideas for new products? A. R&D efforts B. licensing agreements C. consumer research D. geodemographic segmentation E. brainstorming

59. D - All of these but geodemographic segmentation are common sources of new product ideas

90. The local auto supply store gets merchandise delivered to it by its manufacturers as soon as it has a need, reducing stockouts with minimal inventory. This demonstrates the concept of A. data warehousing. B. a push marketing strategy. C. vendor-managed inventory. D. manufacturer-managed distribution. E. cross-docking.

90. C - In vendor-managed inventory systems, the manufacturer replenishes inventories in quantities that meet the retailer's immediate demand, reducing stockouts with minimal inventory.

96. Salespeople should be evaluated and rewarded only for those activities and outcomes that A. generate the most income. B. increase the number of customers. C. fall under their control. D. reduce manufacturing costs. E. cut into competitors' sales.

96. C - It is important to ensure that any measures used to evaluate salespeople focus on factors they can control. For example, a lucky break with which the sales rep had no involvement should not grant the rep a huge commission.

98. A small office supply company may have a person whose primary responsibility is to process routine orders, reorders, or rebuys of products for clients. This employee is known as a(n) A. order taker. B. order getter. C. sales support rep. D. sales team. E. sales manager.

98. A - An order taker is a salesperson whose primary responsibility is to process routine orders for products.

78. Golf ball manufacturers use "Iron Mike," a machine that swings a golf club at a constant velocity, to test the distance for new golf ball designs. When using Iron Mike, the manufacturers are engaged in A. concept testing. B. market testing. C. premarket testing. D. product development. E.alpha testing.

78. E - Iron Mike is a form of alpha testing because it is done within the company, taking the place of a human who might hit the ball repeatedly.

78. The attitudes that Ryan's customers develop after they have purchased his yearly lawn care service will become the basis for determining whether they renew each year. He needs to demonstrate the ____________ dimension of service quality by showing consistently well-maintained yards with neat, professional personnel. A. reliability B. responsiveness C. assurance D. empathy E. tangibles

78. E - The appearance of the yards and of the service personnel refers to the tangibles dimension of service quality

79. One reason auto companies spend millions on racing cars is that they offer the opportunity to test new designs and technology under extreme conditions. Using the cars in a real-use setting provides an opportunity for _______ the product. A. pretesting B. beta testing C. test marketing D. alpha testing E. concept testing

79. B - A beta test is a test of the product conducted by key customers. Participation in NASCAR and other racing organizations offers auto companies a chance to do this.

Differences in weather and climate create opportunities for A. concentrated targeting. B. geographic segmentation. C. benefit segmentation. D. psychographic segmentation. E. demographic segmentation.

B - Climate and weather are two factors that create geographic differences and thus opportunities to segment a market by geographic region

Lionel is asked to conduct an STP analysis for his firm. The first step he should perform in this analysis is to A. develop a business mission statement. B. choose the best target markets. C. reposition existing segments. D. divide the marketplace into subgroups E. conduct a SWOT analysis.

D - The first stage of an STP analysis is segmentation, which involves dividing the market into subgroups. Business mission development and SWOT analysis take place before STP analysis starts, and targeting and positioning are later stages in STP analysis

The first step in the personal selling process is the preapproach

F - The first step is generating and qualifying leads

A value proposition compares the price of a product to its benefits.

F - The value proposition identifies unique benefits or points of difference from competitive offerings that match up with customer needs and wants

Garage sales and online classified ads are examples of C2C marketing

True - These are C2C marketing scenarios, where consumers market to one another.

Jobs in sales are among the highest-paying careers for college graduates

T - In addition to salary, compensation may include perks, commissions, and bonuses.

A roofing company agreed to complete a job in one week and collected a 50 percent deposit, but never showed up to do the job. The same roofing company then donated $6,000 to a local children's hospital. The roofing company could be considered socially responsible.

T - It is possible to be socially responsible yet unethical, which describes this particular company. Its failure to meet its agreements demonstrates ethical issues; however, its community support shows social responsibility

100. If a firm in a purely competitive market can differentiate its product or service, it becomes part of a(n) _______ market. A. pure competition B. oligopolistic competition C. monopolistic competition D. monopoly E. duopoly

100. C - The difference between pure competition and monopolistic competition is that in pure competition, products are not differentiated, but in monopolistic competition, they are.

101.Johnny works at an electronics store. In addition to his salary, he receives 2 percent of the sales dollars he brings in each month. This extra 2 percent is called a A. commission. B. salary. C. bonus. D. reward. E. rebate.

101. A - A commission is a financial incentive, paid based on a percentage of sales volume or profitability.

101. In the early stages of an ad campaign, the objectives are established. To determine if those objectives have been met, the marketer will A. posttest. B. conduct feedback analysis. C. initiate content analysis. D. pretest. E. arrange for peer analysis.

101. A - Posttesting is the evaluation of the campaign's impact after it has been implemented

102. Effective service recovery efforts can lead to all of the following except A. increased purchase intentions. B. increased positive word of mouth. C. increased customer satisfaction. D. lower levels of satisfaction than prior to the service failures. E. increased dependence on technology to prevent future service failures.

102. E - Effective service recovery can positively influence purchase intensions, positive word of mouth, and customer satisfaction; however, satisfaction may still be lower than it was before the service failure

103. Suppose that Walgreens (a major drug store chain) wants to introduce its own brand of cough medicine that is similar in content and packaging to a national brand, but at a lower cost. What kind of brand would Walgreens be introducing? A. private-label brand B. counterfeit brand C. premium brand D. national brand E. manufacturer's brand

103. A - Private-label brands are products developed by retailers

103. McDonald's introduced a Favorites Under 400 Calorie Menu as part of an attempt to reverse the perception that McDonald's sells only unhealthy food. Suppose that McDonald's, as a follow-up, collects and analyzes social media posts from Facebook, Twitter, and similar sites, hoping to understand whether or not consumer perceptions are improving. This would be an example of A. quantitative research. B. an online focus group. C. sentiment mining. D. neuromarketing. E. an experiment.

103. C - Sentiment mining is the collection and analysis of social media posts in order to gain insights into consumers' thoughts and opinions

104. Gray markets can be a challenge to marketers because A. they are just as illegal as black markets B. they may tarnish the manufacturer's image C. they are legal in some states and illegal in others. D. consumers are against them, but retailers support them E. they may result in price increases across the board.

104. B - A gray market is a situation in which goods end up being sold at prices lower than those intended by the manufacturer. This can hurt the manufacturer's image by communicating poor quality

105. Overriding desires that drive how we live our lives are called A. self-values. B. self-concept. C. self-esteem. D. self-confidence. E. self-control.

105. A - Self-values are goals for life, not just the goals one wants to accomplish in a day. They are the overriding desires that drive how a person lives his or her life.

105.Experts estimate that the average cost of a single B2B sales call is about A. $50. B. $190. C. $600. D. $1,000. E. $1,500.

105. C - The average cost of a single B2B sales call is about $600

106. When travelers are bumped from overbooked flights, they are frequently offered vouchers good for future travel. The dollar value of the voucher is the airline's estimate of A. perishable value. B. distributive fairness. C. empowerment. D. procedural justice. E. the size of the knowledge gap.

106. B - Distributive fairness is the evaluation of fairness of the outcome (i.e., a fair compensation for whatever loss or inconvenience occurred as a result of the service problem)

106. The only formal regulation of children's advertising appears in A. the FCC regulations regarding children. B. the provisions of the Children's Television Act of 1990. C. the Better Business Bureau's Children's Advertising Review Unit guidelines. D. the federal GAO regulations on advertising. E. the Food and Drug Administration guidelines.

106. C - The Children's Television Act of 1990 limits the amount of advertising broadcast during children's viewing hours and is the only formal regulation of children's advertising

107. Because customers have different needs and expectations, the key to distributive fairness in service recovery is to A. listen to the customer. B. contact a supervisor quickly. C. estimate the damage. D. provide a fair solution. E. resolve the problem quickly.

107. A - Distributive fairness is the evaluation of fairness of the outcome (i.e., a fair compensation for whatever loss or inconvenience occurred as a result of the service problem). Only by listening to the customer can the service provider understand what the customer will consider a fair result

107. The head of the marketing area told Alex to find the most stringent federal regulations on advertising to create the toughest standards. "If we can pass those, we should be able to get by all regulations." One real problem is A. state regulations are not always consistent with federal standards. B. FCC regulations create a uniform standard that preempt all other regulations. C. European countries have agreed to follow the U.S. standards, so it's a matter of working with the U.S. federal regulations. D. as long as the advertising does not contain sexually explicit content, there are no regulations. E. in most cases, standards are changing almost daily.

107. A - States have different regulations and are beginning to assert their authority more than in the past, creating confusion since many state laws are more restrictive than U.S. federal law. The same is true of European Union laws regulating advertising—they are typically more restrictive than U.S. law.

107. In the United States, most consumer packaged goods found in grocery and discount stores are in the __________ stage of the product life cycle. A. growth B. pioneer C. introduction D. decline E. maturity

107. E - In the United States, most consumer-packaged goods found in grocery and discount stores are already in the maturity stage.

107. The commercial airline industry is considered what type of market? A. duopoly B. monopoly C. monopolistic competition D. pure competition E. oligopolistic competition

107. E - When a market is characterized by oligopolistic competition, only a few firms dominate.

108. U.S.-based global marketers have often found that A. anything goes when advertising outside the United States. B. EU advertising laws are more restrictive than those in the United States. C. children in other countries are less influenced by advertising. D. self-regulatory groups are especially demanding of U.S. advertisers. E. outside the United States, standards are subject to UN approval.

108. B - Advertising laws in the European Union are more restrictive than those in the United States.

108. Randall arrived at the hotel to find that, although he had a guaranteed reservation, the hotel had no rooms available. He became angry when the hotel made him a reservation at a more expensive hotel but refused to pay the difference in room rates. Randall was upset because, in his opinion, the hotel's solution did not incorporate A. intangible fairness. B. distributive fairness. C. procedural fairness. D. service fairness. E. empowerment fairness.

108. B - Distributive fairness is the evaluation of fairness of the outcome (i.e., a fair compensation for whatever loss or inconvenience occurred as a result of the service problem). In this case, Randall did not feel that he was compensated for the additional cost he had to bear due to the problem with his reservation

108. What is the best situation in the Circles for a Successful Value Proposition framework? A. The firm's benefits overlap with both customer needs/wants and competitors' benefits. B. The firm's offering overlaps with customer needs/wants but not with competitors' offerings. C. The firm's offering overlaps with competitors' benefits but not with customer needs/wants. D. Customer needs/wants overlap with both the firm's benefits and competitors' benefits. E. None of the three circles overlaps.

108. B - The best situation is if a firm's product or service offering overlaps with customer needs and wants but suffers no overlap with competitors' offerings.

108. Everyday low pricing (EDLP) provides value to consumers by A. continually offering items on sale B. minimizing the number of options a consumer can evaluate C. offering noncumulative quantity discounts. D. reducing their search costs. E. creative use of reference pricing

108. D - Everyday low pricing focuses on providing reasonable prices all the time on products, instead of using sale items and regular, higher prices. This allows consumers to shop in one place and know they are getting a reasonable price, instead of having to search for bargains

108. Telemarketing and cold calls have become less popular as sales tools because of all of the following reasons except A. their success rate is low. B. they can be expensive. C. they are impacted by state and federal laws prohibiting them under certain conditions. D. during cold calls, the salesperson is not able to establish the customer's needs ahead of time. E. they require appointments, which takes time away from the actual selling of the product.

108. E - Neither cold calls nor telemarketing calls require appointments. The rest of the statements are true

109. How is consumer panel data collected during the test marketing phase of a new product introduction? A. Panelists complete a questionnaire about the product. B. Panelists scan their receipts on a home scanning device. C. Stores provide sales data to companies. D. Consumers voluntarily go to a website to record their preferences. E. Focus groups are held in key markets.

109. B - The consumer panel data are collected by panelists scanning in their receipts using a home scanning device

109. Most customers want to achieve a fair solution following a service failure. Which of the following is not a factor that affects a person's perceptions of "fairness" in these kinds of situations? A. the nature or severity of the service failure B. the customer's experience with other firms C. the firm's policy on service recovery D. observed treatment of other customers E. stories of service recovery told by friends and family

109. C - Customers do not care much about firm's policies, if these policies seem unreasonable. The other factors, though, will affect customers' perceptions of fairness

109. In the Circles for a Successful Value Proposition framework, the value proposition is represented by A. the intersection between customer needs/wants and competitors' benefits. B. the intersection between the firm's benefits and competitors' benefits. C. the intersection between customer needs/wants and the firm's product's benefits. D. the intersection between all three of the circles. E. the part of the firm's benefits that doesn't overlap with anything.

109. C - The value proposition is the intersection between customer needs/wants and the benefits offered by the firm. This is the area in which customer needs/wants match the benefits the firm can offer

110. Tomas, a bank employee, doesn't feel that his coworkers accept him. He decides to dress more casually, as they do, hoping to be accepted. Which level of Maslow's hierarchy of needs is Tomas trying to work on? A. esteem B. physiological C. safety D. love E. self-actualization

110. D - The five groups are physiological, safety, love, esteem, and self-actualization. Tomas seeks acceptance from his colleagues, which represents the love level

110. In the Circles for a Successful Value Proposition framework, the portion of the customer needs/wants circle that doesn't overlap with anything else represents A. the firm's value proposition. B. competitors' value propositions. C. low-priority needs and wants that customers are willing to give up. D. unmet customer needs/wants. E. unknown customer needs/wants that the firm does not understand.

110. D - The portion of the customer needs/wants circle that doesn't overlap with anything represents customer needs/wants that are at present unmet by any offering. These represent an opportunity to serve the customers by meeting these needs.

111. What makes a high/low pricing strategy appealing to sellers? A. It attracts two distinct market segments. B. It allows the seller to market itself as an "everyday low price leader." C. It doesn't require the seller to continually offer sales or deep discounts D. It allows sellers to capture the most profit from a new product or service E. It reduces the need for slotting and advertising allowances

111. A - A high/low strategy is appealing because it attracts two distinct market segments: those who are not price sensitive and are willing to pay the "high" price and more price-sensitive customers who wait for the "low" sale price. High/low sellers can also create excitement and attract customers through the "get them while they last" atmosphere that occurs during a sale.

111. There are five types of risks associated with purchase decisions. Which of the following best describes a situation where your new car stalls in the middle of a busy intersection? A. physiological risk B. social risk C. financial risk D. functional risk E. psychological risk

111. A - Physiological risk (or safety risk) refers to the fear of actual harm, as could happen in this situation. This might also be considered an example of performance risk, since the product has failed, but it was not one of the options offered.

111. Cathy has used Olay beauty products for years, even though there are many competitors on the market. She plans to purchase a recently introduced micro-sculpting cream from Olay's Regenerist line. Olay is in the _________ stage of the product life cycle. A. introduction B. growth C. evaluation D. maturity E. decline

111. D - Although P&G does come out with new variations of the Olay product, it is in the maturity stage as P&G must defend its market share against fierce competition

111. Which of these is not one of the main components of a value proposition? A. the target market B. the name of the product or brand being offered C. the product/service category or concept being offered D. the price of the product or service being offered E. the unique point of difference or benefits offered

111. D - The value proposition consists of a target market, a brand/product name, a category or concept, and a unique point of difference or benefit. The price might be the unique point of difference in some cases, but otherwise price does not need to be part of the value proposition

112. Nadia stood outside the mall and asked people which stores they visited and if they bought anything. If they said yes, she asked them what they bought and how they came to the decision to buy that item. What form of research was Nadia most likely conducting? A. questionnaire B. in-depth interviews C. experiment D. focus group E. biometrics

112. B - In an in-depth interview, trained researchers ask questions, listen to and record the answers, and then pose additional questions to clarify or expand on a particular issue.

112. According to your text, _______ packaging is product packaging that is ecologically responsible. A. recycled B. environmental C. responsible D. sustainable E. green

112. D - An interesting recent development in packaging is a move to "sustainable packaging." Sustainable packaging is product packaging that is ecologically responsible.

113.When her salespeople bring in five new customers, Marissa gives them a $1,000 payment. This is an example of a A. commission. B. bonus. C. sales contest. D. nonfinancial reward. E. salary.

113. B - A bonus is a payment made at management's discretion when the salesperson attains certain goals

113. Kevin is recognized by his friends as an activist on many fronts. He prefers to buy products from firms that will donate part of the total purchase price to organizations he supports. Marketers recognize that this approach can be an important competitive tool; it is called A. social activist marketing. B. voluntary premium pricing. C. cause-related marketing. D. business/social responsibility. E. the do-gooder syndrome.

113. C - Cause-related marketing is a commercial activity in which businesses and charities form partnerships to market an image, product, or service for their mutual benefit

113. Firms that are engaged in sentiment mining are analyzing data collected from A. experiments. B. in-depth interviews. C. focus groups. D. social media sites. E. observations.

113. D - Using a technique known as sentiment mining, firms collect consumer comments about companies and their products on social media sites such as Facebook, Twitter, and online blogs. The data are then analyzed to distill customer attitudes toward and preferences for products and advertising campaigns

114. When Meg planned her trip to Australia, she spent months evaluating packages, airfares, and hotel accommodations. Meg's trip is an example of a(n) ________ product. A. specialty B. convenience C. unsought D. shopping E. sought

114. D - Since Meg spent significant time comparing alternatives, this would be considered a shopping product

114.________ training is excellent for communicating selling and negotiation skills, because managers can observe the sales trainees in real selling situations and provide instant feedback. A. Role-play B. Internet-based C. Simulation D. On-the-job E. Distance learning

114. D - This describes on-the-job training programs

114. Charging a relatively high price for new and innovative products to those consumers most willing and able to pay the high price is called price A. penetration B. bundling C. fixing D. referencing E. skimming

114. E - here are two primary new product pricing strategies: skimming, which focuses on selling at a high price to the innovators and early adopters on the diffusion of innovation curve; and market penetration, which focuses on selling at a low price in order to gain market share as quickly as possible.

115. When Apple Computer Company introduced the iPhone—a combination phone, MP3 player, and Internet access device—in 2007, it was priced at $499, considerably higher than either the iPod or competing cell phones. Hoping to attract innovators and early adopters, Apple was most likely pursuing a __________ pricing strategy. A. market penetration B. slotting allowance C. price fixing D. reference price E. price skimming

115. E - In many markets, and particularly for new and innovative products or services, innovators and early adopters are willing to pay a higher price to obtain the new product or service. This strategy, known as price skimming, appeals to these segments of consumers who are willing to pay the premium price to have the innovation first

116. When John checked into his Orlando hotel, the front desk clerk informed John that his room would not be ready for another 20 minutes. John didn't mind waiting; he had arrived well ahead of the standard check-in time and this wait time fell within his __________, the area between his expectations regarding desired service and the minimum level of service he will accept. A. zone of tolerance B. delivery gap C. zone of intolerance D. service gap E. patience zone

116. A - The zone of tolerance is the difference between what a customer really wants and what the customer will accept before going elsewhere.

116. Which of the following is the least interactive IMC strategy? A. personal selling B. mobile marketing C. online marketing via social media D. direct marketing via catalog E. direct marketing via telemarketing

116. D - The elements of an IMC strategy can be viewed on two axes: passive and interactive (from the consumer's perspective) and offline and online. Direct marketing via catalog is considered a passive form.

116. For a price skimming strategy to work, the product or service must A. be bundled with products or services already available on the market. B. be similar to what consumers are already comfortable with. C. have wide market appeal D. have low production costs E. offer consumers some new benefit currently unavailable in alternative products

116. E - For a price skimming strategy to work, the product or service must be perceived as breaking ground in some way, offering consumers new benefits currently unavailable in alternative products.

117. Before deciding on a new promotional campaign, Jeffrey's Muffins looked at its customers' buying patterns over a 15-year period as determined by their use of a loyalty card. What method did Jeffrey's use to make sense of the data that were available? A. biometrics B. neuromarketing C. data mining D. data warehousing E. panel data

117. C - Jeffrey's used data mining to analyze customer buying patterns by analyzing the data that were captured from loyalty cards.

117. What is one of the drawbacks of using a price skimming strategy? A. It is difficult to lower prices on an introductory product B. Price skimming allows competitors to easily enter the market C. Firms must consider the high costs associated with producing a small volume of product. D. Price skimming does not work well with new-to-the-market, innovative products E. Price skimming is often challenged as being illegal

117. C - Skimming strategies face a significant potential drawback in the relatively high unit costs associated with producing small volumes of products. Therefore, firms must consider the trade-off between earning a higher price and suffering higher production costs.

118. With a __________ pricing strategy, marketers set a low initial price for the introduction of a new product or service. A. market penetration B. Bundling C. price fixing D. Reference E. skimming

118. A - Market penetration focuses on selling at a low price in order to gain market share as quickly as possible.

118. As firms become more sophisticated in their communication efforts, the trend is toward company blogs becoming more A. interactive. B. expensive. C. company-controlled. D. consumer-controlled. E. entertainment-oriented.

118. A - Nowadays, blogs are becoming more interactive as the communication between bloggers and customers has increased.

119. When Apple's famous 1984 "Big Brother" ad aired during the Super Bowl, it reached an estimated 500 million viewers. It aired only once on commercial television, but was seen in later years in at least 10 television programs recalling great commercials. For its target audience, the frequency of this ad is measured as A. 1. B. 10. C. 500 million. D. one-tenth. E. 5 billion (10 times 500 million).

119. A - Marketers measure the frequency of exposure—how often the target audience is exposed to a communication within a specified period of time. In this case, the ad was aired only once

119. The major objectives associated with a market penetration pricing strategy are to A. capture the high end of the market demand curve and lower introduction costs B. quickly build sales and market share C. minimize customer dissatisfaction and maximize reference price value D. provide an incentive to purchase a less desirable product to obtain a more desirable product E. match competitors' prices and communicate high quality

119. B - Firms using a market penetration strategy set the initial price low for the introduction of the new product or service. Their objective is to build sales, market share, and profits quickly. The low market penetration price is an incentive to purchase the product immediately

120. ________ can be defined as raw numbers or other factual information that, on their own, have limited value to marketers. A. Statistics B. Charts and graphs C. Marketing research D. Formulas E. Data

120. E - This describes data. When the data are interpreted, they become information, which results from organizing, analyzing, and interpreting data and putting them into a form that is useful to marketing decision makers.

121. The saying "leaving money on the table" is associated with A. a price skimming strategy that forces consumers to choose between products B. a market penetration strategy when there is an opportunity for price skimming C. vertical price fixing in markets where horizontal price fixing would be more appropriate D. a predatory pricing strategy that results in excessive seasonal discounts E. loss leader pricing that drives consumers to competitors' products.

121. B - One problem with a market penetration strategy is that it may offer the product at a low price to customers who would be willing to pay more

121. A firm's marketing communication strategy is formulated specifically to A. increase its frequency ratio. B. control its public image. C. increase its social media presence. D. communicate the value of its product(s). E. increase its return on investment.

121. D - Marketers must consider how to communicate the value of a product and/or service—or more specifically, the value proposition—to the target market. A firm must develop a communication strategy to demonstrate the value of its product

122.__________ is when two or more firms join forces to reach a target audience in a short-term effort. A. Promotional co-branding B. Cross-promotion C. Joint-venture promotion D. Multi-target promoting E. Convenience promoting

122. B - This is the definition of cross-promotion

123. Traditionally, marketers have seen the role of __________ as generating short-term results, whereas the goal of __________ was to lead to long-term results. A. public relations; institutional advertising B. advertising; personal selling C. sales promotion; advertising D. advertising; public relations E. corporate blogs; public relations

123. C - Sales promotion is often viewed as a vehicle to generate short-term results; advertising is often viewed as a vehicle to generate longer-term results, in part because repeated viewing is often required for the consumer to completely process the advertising message

124. Lars wants to purchase a gift for a colleague whose home he will be visiting. He decides to bring luxury chocolates, as he knows most people enjoy them. Although he is not sure about specific products, Lars heads directly for a store selling Godiva Chocolates, because he knows this is a high-end brand. Lars can be said to have A. brand recall. B. aided recall. C. brand preference. D. brand image. E. brand awareness.

124. E - Brand awareness refers to a potential customer's ability to recognize or recall that the brand name is a particular type of retailer or product/service

125. The individual elements of an IMC strategy can be viewed on two axes: __________ (from the consumer's perspective) and ________. A. passive and interactive; tangible and intangible B. mobile marketing and direct marketing; tangible and intangible C. offline and online; low cost and high cost D. passive and interactive; offline and online E. static and changing; offline and online

125. D - The elements of an IMC strategy can be viewed on two axes: passive and interactive (from the consumer's perspective) and offline and online

126. The Clayton Act and the Robinson-Patman Act forbid certain types of A. price discrimination. B. bait-and-switch pricing C. predatory pricing D. everyday low pricing strategies E. loss leader pricing

126. A - When a firm sells the same product to different resellers at different prices, it is considered price discrimination. Certain types of price discrimination are illegal under the Clayton Act and the Robinson-Patman Act.

130. It is the responsibility of __________ to determine the ethical approach to setting prices so consumers find value and the firm can make a profit. A. the Better Business Bureau B. federal regulators C. the American Marketing Association D. marketers themselves E. industry standards boards

130. D - Marketers must be aware of ethical considerations in pricing and should set prices accordingly

23. Paul subscribes to an Internet service that alerts him whenever other firms in his industry are quoted in the media. Paul is using this type of marketing research primarily to A. provide a link between him and his production center. B. help him understand the needs of his customers. C. monitor his competitors. D. increase profits through the sale of syndicated data. E. decide how to price his new products.

23. C - Because Paul's service is informing him of quotes from managers of others in his industry, he is monitoring his competition.

24. By adding new product lines beyond its core business of computer software, like the Surface Pro tablet and Xbox 360 game system, Microsoft primarily benefits by A. keeping up in a market where sales come mostly from new products. B. satisfying the changing needs of the technological research staff. C. avoiding market saturation from products that have just been introduced to the market. D. creating diversification and reducing risk. E. spreading out production costs.

24. D - If Microsoft were totally dependent on software sales, a shift away from Windows would damage its sole market. By diversifying into game systems and tablets, Microsoft seeks to be less dependent on any one market.

25. Through analysis of sales data, Price-Cutters retail store found that customers who bought peanut butter also tended to buy bananas. Price-Cutters was engaged in A. syndicated surveying. B. focus group analysis. C. behavioral analysis. D. data mining. E. structured sampling.

25. D - Data mining consists of using statistical techniques to discover patterns and relationships in company data. In this case, the connection between peanut butter and banana purchases was discovered.

26. Because advertising is the most visible form of marketing, A. many people think of marketing and advertising as synonymous. B. it is the most important part of a marketing mix. C. everyone prefers to be in advertising. D. marketing budgets always emphasize advertising over other forms of communication. E. it is the largest source of employment opportunities for marketing graduates.

26. A - Many people think of advertising when they hear the term marketing because advertising is such a visible form of marketing.

27. As noted in your text, in Arthur Miller's play Death of a Salesman, Willie Loman portrays A. a leading example of an ethical salesman. B. the loneliness of a traveling salesman. C. the ruthlessness of a pushy salesman. D. the lifestyle and success of people in sales. E. the value salespeople provide to consumers.

27. B - Willie Loman represented the dark side of selling, as is the case with many other portrayals of salespeople in popular culture

27. The complete set of all products offered by a firm is called its A. product line. B. product categories. C. product mix. D. product breadth. E. product line depth.

27. C - The product mix is the complete set of all the firms' offerings.

27. In countries like the United States, services A. have almost all been replaced by technology. B. are a small portion of GDP relative to manufacturing. C. are replacing property taxes as a source of government revenue. D. will decrease in demand as the population ages. E. account for an increasing share of GDP.

27. E - The economies of many developed countries are heavily dependent on services. Services account for about 76 percent of the U.S. gross domestic product

28. A __________ is a group of associated items that consumers tend to use together or think of as part of a group of similar products. A. product line B. product mix C. product mix breadth D. line extension E. brand extension

28. A - A product line is a subset of the product mix, a collection of associated items that consumers either use together or consider to be related to each other.

30. Which of the following is based on the principle that everything we need for our survival and well-being depends either directly or indirectly on our natural environment? A. Corporate Social Responsibility B. Sustainability C. Business ethics D. Marketing ethics E. Ethical climate

30. B - According to the U.S. Environmental Protection Agency, "Sustainability is based on a simple principle: Everything that we need for our survival and well-being depends, either directly or indirectly, on our natural environment." Therefore, sustainable actions, including sustainable marketing, allow "humans and nature [to] exist in productive harmony, that permit fulfilling the social, economic and other requirements of present and future generations."

30. In developing marketing strategies, why is price often the most challenging of the four Ps to manage? A. because most managers feel it is the least important element of the marketing mix B. because it is the least understood element of the marketing mix C. because it has to be based on the promotion budget for the product D. because it is difficult to calculate markups for products E. because managers don't understand the relationship between benefits and costs

30. B - Price is the most challenging of the four Ps to manage, partly because it is often the least understood. In addition, despite the complex considerations when setting prices, pricing is often treated as an afterthought.

30. The marketing of services differs from product marketing because services are all of these except A. intangible. B. inseparable. C. heterogeneous. D. renewable. E. perishable.

30. D - All of these are differences between products and services, except for the quality of renewable.

30. When Karen realized her dog had fleas, Karen had A. a social-perceptual incongruence. B. a psychological need. C. a cognitive learning failure. D. a universal shopping need. E. an unsatisfied need.

30. E - Need recognition is the first step in the consumer buying process

31. Carlos owns a chain of retail electronic stores. He is evaluating how he allocates his firm's IMC budget. He receives offers from a variety of advertising media, spends money on his firm's public relations efforts, and is considering electronic media alternatives. Carl must recognize that each IMC alternative A. will stand on its own B. is part of the whole C. is part of noncommercial free speech D. doesn't help communicate value to the consumer E. is dictated by the nature of the supply chain

31. B - An integrated marketing communication strategy will have many components that must be coordinated into an integrated whole.

31. The greater the discrepancy between a consumer's needy state and the desired state, the greater A. time needed to satisfy the need. B. the effort consumers will invest in searching for alternatives. C. the consumer's need recognition will be. D. the size of the universal set will be. E. the amount of external information search will be needed.

31. C - A greater discrepancy will create a stronger drive to satisfy the need

31. Historically, prices were: A. the center of attention in almost all marketing strategies. B. analyzed and changed constantly. C. calculated to minimize contribution per unit. D. allowed to vary seasonally as cross-shopping tendencies fluctuated. E. rarely changed except in response to radical shifts in market conditions.

31. E - Historically, prices did not change much except in response to major changes in marketing conditions.

32. The greater the discrepancy between a consumer's __________, the greater the consumer's need recognition will be. A. financial risk and performance risk B. search for alternatives and alternatives found C. needy state and desired stat D. universal set and evoked set E. external and internal information search

32. C - A greater discrepancy between the two states will create a stronger drive to satisfy the need

32. The three elements of any IMC strategy are the consumer, the channels, and A. the receiver B. the product C. the evaluation of results D. the company E. event sponsorship

32. C - An IMC strategy must consider the consumer, the channels, and evaluation of the results

32. Assuming that a marketing research study will answer important questions and reduce uncertainty associated with the proposed project, a major question that needs to be addressed before starting the study is A. How will the results be presented? B. Will observational research be considered intrusive? C. Is top management committed to the study? D. How will the questions be defined? E. Who will manage the research?

32. C - In addition to ensuring that the research will guide or influence decision making, it is also important to ensure that top management will accept the results of the research. Otherwise, the research is likely to be a waste of time

33. Gary is the marketing manager for an automobile dealership. His boss tells him the firm's primary goal is to increase its local market share from 15 to 30 percent. His firm is using a ________ orientation. A profit B sales C competitive D customer satisfaction E product development

33. B - Gary's pricing strategy is focused on increasing sales to gain market share.

33. As the number of communication media has increased, the task of understanding how best to reach target customers has A. shifted from creating a value proposition to revising a value proposition. B. increased the use of rule-of-thumb targeting C. become easier D. focused on reducing communication noise E. become more complex

33. E - The proliferation of communication media—driven largely by technology—has made it much more difficult to choose the best way to reach a target market.

34. Yana has identified the target audience for her line of Russian jewelry. For optimum success, she is creating an advertising plan that will A. allow her to skip the assessment stage at the end of the campaign. B. offer discounts to media as an incentive to carry out her plan. C. clarify the specific goals that the advertising is designed to accomplish. D. encapsulate her unique selling proposition. E. maximize puffery.

34. C - Specific, measurable objectives are important both in directing campaign strategy and in measuring results.

34. The process by which the use of a new product or service spreads throughout a market group is referred to as A. new product introduction. B. lead user dispersion. C. diffusion of innovation. D. the product life cycle. E. product development diffusion.

34. C - The adoption of an innovative product or service spreads through the population over a period of time and generally follows a bell-shaped curve (called the diffusion of innovation curve). The product life cycle is a related but different concept that traces sales (not initial adoption) of a product throughout its life.

36. A key to successful marketing is determining how to meet the correct balance of __________ needs that best appeals to the firm's target markets. A. functional and socia B. postpurchase and prepurchase C. safety and situational D. psychological and physiological E. functional and psychological

36. E - Marketers must understand what needs are most likely to drive customers to consider their products, and design offerings to meet those needs.

36. Martha had several unpleasant experiences trying to find the merchandise she needed at a large lumber yard and hardware store. The employees—when she could find them—rarely seemed to know where anything was outside of their own departments. But on her most recent visit, she was pleasantly surprised to find that the store had installed kiosks where she could get directions quickly and accurately. The store had found a technological solution to the services marketing issue of A. intangibility. B. part-time employees. C. perishability. D. inseparability. E. heterogeneity.

36. E - Martha has been experiencing problems with service heterogeneity, or variability—the employees are not consistently able to assist her. The kiosks provide a solution by offering comprehensive store information

36. Many executives and corporate boards of directors do not perceive social responsibility as part of their mission or responsibility. These business leaders consider corporate social responsibility to be A. a basic requirement of any business. B. the equivalent of the AMA ethical value "Do no harm." C. the key to operational effectiveness. D. a component of basic business ethics. E. beyond the norms of corporate ethical behavior.

36. E - Some managers believe that firms should focus on increasing shareholder value and need not concern themselves with social responsibility.

37. When a customer purchases a DVD at Best Buy, all of the following information flows in the supply chain are started except A. the sales associate scans the UPC recording the sale B. the sale is transmitted to Best Buy's distribution center to adjust inventory data C. the purchase is added to the customer's purchasing habit records D. Best Buy's buyer aggregates sales at all stores and uses the information to send a reorder to the manufacturer E. the point-of-sale terminal records the sale and sends it to Best Buy's buyer

37. C - Although customer-specific data may be collected at the point of sale, they are not part of the information that flows in the supply chain.

37. Although a sales representative may skip a step in the personal selling process or might sometimes have to go back and repeat steps, there is logic in the sequence. Which of the following would not be appropriate in the personal selling process? A. Before a salesperson can work through the preapproach, leads must be qualified. B. The customer's reservations must be addressed before closing the sale. C. Closing the sale is the final—and most satisfying—part of the process. D. Carefully working through the preapproach will make the next step—the sales presentation—more effective and efficient. E. Follow-up may include additional sales for the representative.

37. C - Closing the sale is not the final part of the process. Postsale support is essential as well for relationship building and to encourage follow-on business.

37. When marketers state that services are ____________, they are referring to the fact that services are not always of the same quality from one time period to another or from one service provider to another. A. intangible B. inseparable C. heterogeneous D. perishable E. viable

37. C - Heterogeneity refers to the problem that services tend to vary by time, location, or the person providing the service. The human element is primarily responsible for this variation

37. Being socially responsible is generally considered A. a good thing to do only if a company is profitable. B. inappropriate for most firms in today's challenging markets. C. going above and beyond the norms of corporate ethical behavior. D. a necessary part of every firm's strategy. E. the responsibility of corporate-sponsored foundations that can effectively concentrate a firm's good deeds.

37. C - While corporate social responsibility programs can benefit society as well as the firm itself, these programs are generally considered "above and beyond" and are not a necessity for every firm.

37. A _______ strategy involves accurately measuring all the factors needed to predict sales and profits at various price levels, so that the price level that produces the highest return can be chosen A. sales orientation B. target profit C. target return D. status quo E. maximizing profits

37. E- A maximizing profits strategy attempts to make the greatest possible profit, and therefore will set prices accordingly. A great deal of information is needed to make this determination.

38. The old restaurant saying "You are only as good as the last meal served" reflects the fact that services are A. intangible. B. perishable. C. heterogeneous. D. portable. E. viable.

38. C - This saying refers to the heterogeneity, or variability, of services—the fact that they tend to vary by time, location, or service provider

38. Steven managed an auto parts store in the 1990s. At that time, stockouts—failure to have the parts auto mechanics needed to do their work that day—increased the likelihood of the mechanics becoming customers of competing auto supply stores. To avoid this problem and keep his business customers, Steven most likely A. canceled orders and shifted the business to competitors B. increased promotional outlays to overcome customer dissatisfaction C. used exclusive geographic territories to reduce supply chain inefficiencies D. stockpiled inventory, adding to the cost of providing auto parts E. returned to traditional distribution center logistics

38. D - In response to supply chain inefficiencies—and particularly stockouts—many retailers in the 1990s stockpiled inventory, leading to higher inventory carrying costs.

39. Monica works as a salesperson in a retail clothing store. Of the five stages in the selling process, Monica is least likely to engage in A. generating and qualifying leads. B. making the preapproach C. closing the sale D. following up E. making a sales presenation

39. A - Monica is least likely to engage in generating and qualifying leads. For retail salespeople, qualifying leads is inappropriate and possibly discriminatory. It is dangerous to "judge a book by its cover," assuming based on someone's appearance that he or she is not qualified to buy products in a store

39. Marketers can take advantage of the variable nature of services by A. merging services with products. B. customizing services to meet customers' needs. C. offering to expedite intangibles. D. expanding the standards gap. E. using strict standardization.

39. B - The positive side of service variability is that it offers a chance to customize delivery. The reason "strict standardization" is not the right answer is because this is a method used to reduce variability, not to take advantage of it

40. Chesnee works in the office of a building materials company. One of her jobs is to identify new building projects and to determine who will make the building materials purchase decisions. Chesnee is involved in the _______________ step of the selling process. A. generate leads B. preapproach C. closing the sale D. follow-up E. sales presentation

40. A - Chesnee is involved in generating leads for her building materials company—finding prospective customers

40. Julia wants her firm's gourmet snacks to be the leading brand in the U.S. market. When adopting a pricing strategy designed to gain market share, she should remember that A. rarely is the lowest price offering the dominant brand in a market B. prestige products need to be competitively priced C. companies can gain market share by offering low-quality products at a high price D. total value equals total cost minus variable costs leading to price escalation E. price wars are the way to become the dominant brand

40. A - Julia is seeking market share, not profits, but she should bear in mind that market leaders are rarely the lowest-priced products in the market.

40. One approach marketers are using to reduce service __________ is to replace people with machines whenever appropriate. A. intangibility B. inseparability C. spendability D. perishability E. heterogeneity

40. D - By replacing people with technology, service delivery tends to become more standardized, which reduces the variability, or heterogeneity, of delivery. Of course, this assumes that the machines don't break down!

41. Several months ago, Veronica took over the sales territory managed by a very successful salesperson. Veronica should use __________ as a first source of leads. A. current customers B. chamber of commerce gatherings C. trade shows D. census data E. competitor databases

41. A - Current customers should be a good source of leads, given the success of her predecessor. They may be able to provide referrals to their business associates.

41. One key feature of the value of a strong brand is that A. it can protect the firm from competition. B. it no longer needs to be supported by advertising and promotion. C. if it becomes a generic name, the brand is worth even more. D. it cannot be successfully imitated by a retailer's own brand. E. competitors will typically abandon a sector altogether rather than compete.

41. A - Strong brands protect the firm somewhat from competition because the brand differentiates the firm's products, making customers more loyal

41. For new product marketers, early adopters are important because they tend to be A. few in number. B. the first to adopt a new product. C. opinion leaders. D. fond of prototypes. E. alpha testing enthusiasts.

41. C - Early adopters, who are not the very first buyers of a new product (those are the innovators) but who still buy very early in the product's life, are important to the diffusion of the products because they are often opinion leaders

42. When consumers associate a brand with a certain level of quality and familiar attributes, allowing consumers to make quick decisions, the brand A. establishes novelty. B. is the only one the consumer will consider. C. reduces delivery costs. D. facilitates purchasing. E. creates brand dilution.

42. D - A brand can facilitate purchasing because the customer can quickly recognize it and decide whether or not to purchase it based on brand associations

43. Psychographics is the segmentation method that delves into how consumers A. adjust to demographic changes. B. allocate scarce incomes to a variety of goods and services. C. describe themselves. D. value their livelihoods. E. believe other people see them.

43. C - Psychographics is related to the way we describe ourselves—our self-concept, values, and lifestyle.

43. Company sales invoices, census data, and trade association statistics are examples of A. primary data. B. data mines. C. secondary data. D. tertiary data. E. qualitative data.

43. C - These are all examples of secondary data—data that were collected for another purpose but that can be helpful in answering questions. Some secondary data can come from internal sources such as a company's sales invoices, while other secondary data can come from external sources such as census data and trade association statistics.

43. Which of the following is NOT an element of the factors affecting consumers' search processes? A. perceived benefits versus perceived costs of search B. locus of control C. actual risk D. perceived risk E. functional versus psychological need

43. E - Functional vs. psychological needs are categories of needs that can be recognized. The remaining factors influence information search processes.

44. When firms set prices similar to those of competitors, they are following a strategy of A. me-too pricing B. copycat pricing. C. competitive parity D. market-broadening pricing. E. industry-standard pricing

44. C - A competitive parity strategy is a type of competitor-oriented strategy in which the firm sets prices similar to those of the competition.

44. In vendor-managed inventory systems, A. corporations send information to retail customers, bypassing wholesalers and retailers B. companies send information to cooperatives C. customers send information to retailers. D. sales reps send sales information to the retailer E.retailers send sales information to the manufacturer

44. E - Vendor-managed inventory systems transmit store sales and inventory information to manufacturers so that they can reorder needed merchandise

45. Jaleel is the marketing manager for a moderately well-known rock band. He wants to know more about industry trends including sales by different musical styles, online downloads, and concert attendance. Jaleel will most likely use __________ to gather this type of data. A. focus groups B. observation C. syndicated data D. sales invoices E. census data

45. C - Of the options listed, syndicated data are the best choice. Jaleel can look for a data provider who can supply industry sales information. Focus groups and observation are research methods that would not offer comprehensive sales data. Sales invoices could summarize results for Jaleel's band but not for the industry, and census data do not provide the needed information

45. As soon as she saw one, Hillary wanted a flat-screen television, but she was worried about making the wrong choice. She waited until there were alternatives in the market with lower prices and improved quality. Hillary is part of the __________ diffusion of innovation group. A. innovator B. laggard C. late majority D. early majority E. early adopter

45. D - Hillary has waited for a more competitive market with lower prices; this occurs when the early majority is adopting the product.

46. Kathy has naturally curly hair and has often been disappointed with the haircuts she has received. When she moved to a new town, she approached her new office mates and several strangers with curly hair and asked them where they had their hair cut. She chose to spend considerable effort finding a new hair stylist based on the __________ associated with her purchase decision. A. evoked set B. reference group C. physiological risk D. performance risk E. financial risk

46. D - Kathy is afraid of getting another bad haircut. This represents performance risk, in that Kathy is worried about the quality of the service she will receive

46. A __________ brand is one where there is a contractual arrangement between firms allowing one to use its brand name for a fee. A. franchise B. joint venture C. shared D. common use E. licensed

46. E - This is the definition of a licensed brand. A long-standing staple of licensing has been major league sports teams that play in the NBA, NFL, or NHL as well as various collegiate sports teams

47. The value of a brand is often calculated by assessing the A. difference between brand equity and brand liability. B. corporate profitability divided by the monthly brand earnings. C. earning potential of the brand over the next 12 months. D. effect of brand dilution if it occurred. E. average product line depth.

47. C - Earnings potential over the next year offers a rough estimate of a brand's value.

48. Beverage companies often use television commercials with images of young people laughing and enjoying themselves on a beach or in a club. These images are designed to appeal to consumers' __________, suggesting "be like me." A. geographic choices B. self-concepts C. loyalty references D. benefit perceptions E. demographics

48. B - Self-concept is the way we see ourselves. Beverage companies hope that consumers will identify with the ads, seeing themselves (or the way they wish to be)

48. Brand __________ is the set of assets and liabilities linked to a brand that add to or subtract from the value provided by the product or service. A. positioning B. licensing C. association D. equity E. solvency

48. D - This is the definition of brand equity

49. When the delivery of a service fails to meet customers' expectations, a __________ gap exists. A. service B. quality C. standards D. production E. procedural

49. A - The Gaps Model identifies four service gaps: the knowledge, standards, delivery, and communication gaps. In any given situation, one or more of these service gaps may be causing deficiencies in delivered service

49. One reason B2B salespeople spend considerable time qualifying potential customers is that A. they want to have absolutely everything in order before approaching a potential customer. B. it can be costly to prepare and make a presentation to a business customer. C. too many business buyers at trade shows are really people from competing firms trying to obtain competitive information. D. independent agents get the best leads; the company sales representatives need to work harder. E. they want to determine if telemarketing is required.

49. B - A B2B sales presentation can be expensive to prepare and deliver; B2B sales reps want to ensure that the potential customer is a good enough prospect to be worth the expense and effort.

49. A strategy of setting prices based on how customers develop their perceptions of value can often be the most effective pricing strategy, especially if the strategy A. leads the marketer to being the low-cost seller B. is supported by consistent advertising and distribution strategies C. challenges consumers to discard their perceptions of value D. is consistent with a competitive target return strategy E. is measured against the competition

49. B - The question describes a customer orientation in pricing, which can be effective since it is based on value, as long as other aspects of the marketing mix support the strategy.

49. Advertising agencies often provide the IMC function of comparing their customer's target audience with the viewer, listener, or reader profile of the communication channel being considered. The agency is most likely trying to avoid noise problems associated with A. competing messages B. lack of clarity in the message C. a poor choice of medium D. an extended feedback loop E. a flaw in the medium

49. C - Interference in the IMC process is referred to as noise. It can be caused by competing messages, a lack of clarity in the message, or a flaw in the medium. In this case, the choice of the wrong channel can cause most members of the target market to fail to receive the message

51. Firms can close the __________ gap by matching customer expectations with actual service through use of marketing metrics. A. seniority B. knowledge C. standards D. delivery E. communication

51. B - The knowledge gap represents the difference between customer expectations and management's understanding of those expectations. Marketing research is one useful technique for closing this gap

51. The diffusion of innovation theory is useful to marketers in helping them A. adjust to the performance life cycle. B. avoid the cost of concept testing. C. predict which types of customers will buy their product immediately and later. D. predict how long it will take for a new product to gain market acceptance. E. determine a target market.

51. C - The diffusion of innovation theory sheds light on the types of customers who buy products during different stages. It does not predict exactly when these stages in the diffusion process will occur, nor does it help determine a target market.

51.The California Raisin Advisory Board used to run ads featuring "The California Raisins," a fictitious R&B musical group composed of dancing raisins that boogied to the song "I Heard It Through the Grapevine." The ads, which were hugely popular, and which gave rise to a line of raisin-themed merchandise, were designed to encourage raisin consumption. This campaign was an example of A. product-focused advertising. B. consumer-generated advertising. C. public service announcements. D. institutional advertising. E. push strategies.

51. D - This is an example of institutional advertising, which promotes an entire industry (raisin production in California) rather than any particular brand

52. A __________ gap is the difference between the firm's service standards and the service it provides to customers. A. seniority B. knowledge C. standards D. delivery E. communication

52. D - Due to the human element, there is almost always variability in service delivery, in which service sometimes fails to meet the firm's standards. This is the delivery gap.

52. Denny is considering the question, "Did our actions have a negative impact on any stakeholder group?" Denny is addressing marketing ethical issues in the __________ phase of the strategic marketing planning process. A. planning B. control C. implementation D. brainstorming E. situation analysis

52.B - This question evaluates the outcome of marketing decisions, which takes place in the control phase.

53. Every year, General Mills issues a report discussing how the firm has performed against its own standards of ethical conduct. This report is part of General Mills' __________ phase of its strategic marketing planning process. A. planning B. implementation C. control D. evolution E. marketing mix

53. C - General Mills is evaluating its performance, which is a part of the control phase

53. A __________ gap can be closed by getting employees to meet or exceed service standards. A. seniority B. knowledge C. standards D. delivery E. communication

53. D - Due to the human element, there is almost always variability in service delivery, in which service sometimes fails to meet the firm's standards. This is the delivery gap. One way to close the gap is to offer incentives to encourage employees to meet or exceed the standards

54. If there is a difference between the firm's perceptions of customers' expectations and the service standards the firm has set, a __________ gap exists. A. seniority B. knowledge C. standards D. delivery E. communication

54. C - A lack of care in setting specific service standards can lead to a standards gap, a discrepancy between the firm's perceptions of customer expectations and the standards it sets for performance

54. Marketers particularly want their brands and products to be in consumers'__________ sets. A. universal B. retrieva C. evoked D. deterministic E. behavioral

54. C - The universal set represents all brands, regardless of whether the consumer is aware of them. The retrieval set represents all brands the consumer can recall. The evoked set represents all brands the consumer would consider buying. Marketers want their brands to be in evoked sets, since this gives those brands a serious chance of being selected for purchase

55. Marketers that include ethical statements in the firm's mission or vision statements are introducing ethics at which stage of the strategic marketing planning process? A. implementation phase B. control phase C. planning phase D. evaluation phase E. idea generation phase

55. C - During the planning phase, marketers will decide what level of commitment to its ethical policies and standards it is willing to declare publicly. It is during this phase that ethical statements in the firm's mission or vision statements can be incorporated.

55. Elena is in the process of buying a new car. There are many possible cars to choose from, but she is focused on a few she would actually consider buying. These make up her __________ set. A. universal B. retrieval C. immediate D. evoked E. focus

55. D - The universal set represents all brands, regardless of whether the consumer is aware of them. The retrieval set represents all brands the consumer can recall. The evoked set represents all brands the consumer would consider buying (as in this case).

56. The _______ is the ratio of the percentage change in quantity demanded to the percentage change in price. A. cross-price elasticity of demand B. price elasticity of demand C. income elasticity of demand D. break-even point E. demand curve

56. B - Price elasticity of demand measures how changes in price affect the quantity of the product demanded. Specifically, it is the ratio of the percentage change in quantity demanded to the percentage change in price

56. When the marketing research problem is not clearly defined, a researcher will likely engage in __________ research. A. data mining B. qualitative C. survey D. quantitative E. experimental

56. B - Qualitative research can be conducted to investigate the research problem in order to gain preliminary insights; it provides initial information that helps the researcher more clearly formulate the research objectives.

56. During the __________ phase of the strategic marketing planning process, marketers utilize systems to check whether each potential ethical issue raised in earlier phases was addressed. A. implementation B. control C. planning D. assessment E. social responsibility

56. B - The firm assesses its performance regarding ethics during the control phase.

56. The sender of an integrated marketing communication A. must work with the advertising specialists to ensure all recipients interpret the message accurately B. can assess the manner in which receivers interpret the message through gross rating points C. has little control over what meaning any individual receiver will take from the message D. controls the meaning all receivers take from the message E. should attempt to control how the message is received

56. C - The sender cannot control the way a message is decoded. This process is entirely in the hands of the receiver.

56. Regardless of the objective of an advertising campaign, each campaign's objectives must be A. sincere and emotional. B. consistent with those of the available media. C. either informative or persuasive but not both. D. specific and measurable. E. designed for use in both a pull and a push strategy.

56. D - Objectives must be specific and measurable, both to guide campaign development and to facilitate evaluation.

56. If there is a difference between the actual service provided to customers and the service the firm has promoted, a __________ gap exists. A. seniority B. knowledge C. standards D. delivery E. communication

56. E - A communication gap exists if the firm does not keep its service promises.

57. Recently, the number of students enrolled in the marketing program dropped while enrollment in the psychology program increased. The chair of the marketing department will probably use __________ as a first step to gain a better understanding of why enrollments are changing. A. experimental research B. data warehousing C. syndicated data D. qualitative research E. structured questionnaires

57. D - Qualitative research can be conducted to investigate the research problem in order to gain preliminary insights and more clearly formulate the research objectives

58. Which of the following is not a major consideration in determining an advertising budget? A. role of advertising in overall promotional objectives B. the product life cycle C. the nature of the market D. the budgeting method used E. the nature of the product

58. D - In determining an advertising budget, firms must consider the role that advertising plays in their attempt to meet overall promotional objectives, expenditures over the course of the product life cycle, the nature of the market, and the nature of the product

59. When Toyota owners began to report problems with sticking accelerator pedals and nonfunctioning brakes, Toyota at first ignored or rejected the claims. Eventually, the company evaluated the issue and issued a recall. Which of the four steps of the ethical decision-making framework was Toyota performing when it recalled several of its 2007-2010 models? A. assess risk B. identify issues C. gather information and identify stakeholders D. brainstorm and evaluate alternatives E. choose a course of action

59. E - When using the ethical decision-making framework, the first step is to identify the issues. The next three steps are to gather information, brainstorm/evaluate alternatives, and finally to choose a course of action. Toyota, in issuing its recall, had chosen a course of action.

60.The beginning of the sales presentation may be the most important part of the selling process, because this is where the salesperson establishes A. where the customer is in the buying process. B. how much time has been allocated for the presentation. C. which of the alternative products to demonstrate. D. whether to quote a full price or discount price. E. which type of follow-up will be needed.

60. A - Early in the sales presentation, the sales rep needs to determine where the customer is in the buying process by asking a series of questions.

60. A study found that among addicted smokers, a 10 percent increase in the price of cigarettes resulted in a 2 percent decrease in quantity demanded. For these consumers, cigarettes have a(n) _______ price elasticity demand. A. elastic B. inelastic C. cross-price D. income effect E. substitution effect

60. B - If a 10 percent price increase results in a 2 percent reduction in demand, price elasticity of demand is equal to -2 ÷ 10 = -0.2. Price elasticity greater than -1.0 reflects inelastic demand.

60. Which of the following is true about quantitative research? A. It is always conducted using primary data collection. B. It revises the research objective based on data mining. C. It confirms insights and provides a basis for taking a course of action. D. It offers conclusions that are always correct. E. It includes focus group interviews.

60. C - The purpose of quantitative research is to confirm insights and provide a basis for managerial decision making and action.

60. In the product development process, what takes place between concept testing and market testing? A. securing financial backing B. conducting marketing research C. brainstorming D. determining potential ROI E. performing product development

60. E - The third stage of the product development process, between concept testing and market testing, is the actual development of the product.

61. Especially in the fashion industry where styles and trends change rapidly, a quick response system can A. allow retailers to better forecast long-term demand B. reduce logistical overlay C. increase cross-docking promotional effectiveness D. align deliveries more closely with actual sales E. allow manufacturers to introduce unpopular styles and still be successful

61. D - A quick response system in the fashion industry allows smaller, more frequent deliveries of clothing that are matched to recent sales. In this way, as trends change, the retailer will not have a store full of older styles, but can instead adapt quickly to changing consumer preferences.

61. The purpose of __________ is that it confirms insights and provides a basis for taking a course of action. A. primary data collection B. data mining C. qualitative research D. quantitative research E. statistical sourcing

61. D - The purpose of quantitative research is to confirm insights and provide a basis for managerial decision making and action.

61.Like any effective salesperson, Frazer walks into a customer's office, shakes hands, looks the customer in the eye, and smiles. After exchanging pleasantries, Frazer will immediately try to create interest in his company's product and establish A. which type of follow-up will be needed. B. how much time has been allocated for the presentation. C. which of the alternative products to demonstrate. D. whether to quote a full price or discount price. E. where the customer is in the buying process.

61. E - Early in the sales presentation, the sales rep needs to determine where the customer is in the buying process by asking a series of questions.

62. Retail store chains often use __________ data to tailor each store's assortment to the preferences of the local community. A. geodemographic B. benefit C. psychographic D. loyalty E. geographic

62. A - Geodemographic segmentation can be particularly useful for retailers, because customers typically patronize stores close to their neighborhood. Thus, retailers can use geodemographic segmentation to tailor each store's assortment to the preferences of the local community.

62. Anupam's company manufactures industrial ladders. He is concerned that consumers who do not understand ladder safety will purchase these extra-tall ladders and injure themselves. During which phase of the strategic marketing planning process should this issue be addressed? A. control B. planning C. implementation D. design E. ethics

62. C - During the implementation phase, the company can discuss targeting decisions and implementing the marketing mix appropriately to minimize this risk. Perhaps safe use of the ladder should be emphasized in marketing communications, or perhaps steps should be taken to discourage certain consumers from buying these ladders.

62. When consumers are __________, observation becomes particularly useful in understanding consumers' preferences. A. unwilling to participate in research B. confused by questionnaires C. unable to articulate their experiences D. interviewed too frequently E. highly educated

62. C - Observation is especially helpful if people are unable to explain their experiences and preferences. Instead of asking them to think about these issues, people's actual behavior can be observed

62. Marketers are particularly interested in postpurchase behavior because it A. involves both compensatory and noncompensatory consumers. B. offers insights into information search methods. C. avoids situational conflicts. D. involves actual rather than potential customers. E. involves both actual and potential customers.

62. D - Unlike prepurchase behavior, postpurchase behavior reflects the characteristics of actual customers. Satisfied customers may be a source of future purchases and/or positive word of mouth.

63. Sometimes consumers have second thoughts after buying goods that are expensive, infrequently purchased, or associated with a high level of risk. This is an especially critical time for marketers, as their customers are dealing with A. criteria reevaluation. B. buyer's remorse. C. competitive leverage. D. purchase uncertainty. E. consumer vulnerability.

63. B - Buyer's remorse, or postpurchase cognitive dissonance, is felt most often for expensive products purchased infrequently, and/or associated with high levels of risk

64. Students regularly seek out Professor Guillory to advise them. She has an exceptional manner, and students are confident in her and trust her advice. For these students, __________ is the most important of the five service quality dimensions. A. assurance B. reliability C. tangibles D. responsiveness E.empathy

64. A - Assurance is the dimension of service quality that refers to inspiring trust and confidence in others, which is Professor Guillory's strength. Note that the question doesn't actually say that her advice is always good (which would refer to reliability), but rather that students believe that her advice is good.

65. Gerald's Tire Service provides each employee with a clean, sharp-looking uniform. It also instructs employees to put all tools back where they belong and keep the work area clean and uncluttered. Gerald's Tire Service emphasizes __________ in the five service quality dimensions. A. assurance B. reliability C. tangibles D. responsiveness E. empathy

65. C - Gerald's is focusing on tangibles, which are the factors that can be seen, touched, or owned—in this case, uniforms and the cleanliness of the work area

65. How might a technology company like Apple most likely ensure that it behaves in a socially responsible way toward its employees? A. It can pay at least minimum wage when the law requires it. B. It can adhere to government-mandated safety standards in the workplace. C. It can ensure that pay practices are fair at all levels of the company. D. It can ensure that its packaging materials are recyclable. E. Social responsibility isn't relevant where employees are concerned; they are paid for their work and that's enough.

65. C - Obeying the law when it comes to minimum wage and safety isn't sufficient to be considered socially responsible—the firm is simply behaving legally. Using recyclable materials would be a socially responsible practice, but it serves society (the environment), not specifically employees. By ensuring fair pay practices, though, Apple would be treating its employees in a socially responsible manner

65. Postpurchase cognitive dissonance is especially likely for products that are A. cheap, poorly made, and made of plastic. B. personally valuable, antique, or foreign-made. C. simple, easily copied, and new. D. psychologically soothing, purchased impulsively, and part of a consumer's evoked set. E. expensive, infrequently purchased, and associated with high levels of risk.

65. E - Buyer's remorse, or postpurchase cognitive dissonance, is felt most often for expensive products purchased infrequently, and/or associated with high levels of risk

66. One of the reasons marketers use loyalty segmentation is A. the high cost of finding new customers. B. government tax incentives. C. accounting difficulties associated with identifying new customers. D. rapid population increases. E. the failure of micromarketing as a workable strategy.

66. A - It is generally much more expensive to find a new customer than to keep an existing one; thus, it is in the best interests of a firm to retain current customers. For this reason, many firms develop loyalty segmentation-based marketing mixes

67. A systematic ____________ program collects customer inputs and integrates them into managerial decisions. A. quality gap analysis B. empowerment C. zone of tolerance D. standards analysis E. voice-of-customer

67. E - A voice-of-customer program seeks to incorporate customer input into management decision making in a systematic way.

67. In a(n) __________ marketing channel, none of the participants have any control over the others A. cooperative B. corporate C. contractual D. administered E. conventional

67. E - This is the definition of a conventional, or independent, marketing channel

68. Ron sells commercial-grade tools to building contractors. Whenever he visits his customers, he looks to see if they are using his tools for other tasks or have modified the tools for some other purpose. These customers are __________, who can provide ideas for new and improved products. A. laggards B. market testers C. lead users D. reverse engineers E. concept testers

68. C - Ron is looking for lead users, who modify existing products according to their own ideas to suit their needs

68. Sales of national brands of orange juice tend to increase when the economy is doing well, while sales of generic orange juice increase when the economy is not doing well. This is an example of how _______ impacts demand for products. A. dynamic pricing B. the price inelasticity coefficient C. the income effect D. the target return effect E. cross-price elasticity

68. C - This is an example of the income effect, where demand tends to shift from lower-priced to higher-priced products as consumers' income increases, and vice versa. In this case, generic orange juice costs less than do the national brands.

68. Bank of America uses a complex polling system coupled with a customer response measurement system to assess consumers' responses to new products and services. Bank of America is using a(n) __________ program to improve service quality and service offerings. A. quality gap analysis B. empowerment C. zone of tolerance D. standards analysis E. voice-of-customer

68. E - Bank of America is using a voice-of-customer program, which seeks to incorporate customer input into management decision making in a systematic way

69. Randall wants to do an online survey of college professors about the factors that influence their textbook selection. He would like to use a structured survey but is not sure what responses to include for each question. Randall could use __________ to help him develop his survey. A. in-depth interviews B. experiments C. surveys D. observation E. primary data mining

69. A - In-depth interviews are a good way to collect detailed opinions from a small number of individuals

69. __________ are customers who modify existing products according to their own ideas to suit their specific needs. A. Early adopters B. Market testers C. Lead users D. Reverse engineers E. Concept testers

69. C - Lead users, who modify existing products to meet their needs, are a particularly good source of new product ideas

69. In the movie Field of Dreams, one of the memorable phrases is, "If you build it, he will come." The AIDA model would suggest that after marketers "build" a product or service and create awareness among consumers, they need to A. determine the level of desire needed to sustain action. B. promote sufficiently to gain global attention. C. create interest among consumers, persuading them to investigate further. D. offer discounts to increase purchase intentions. E. take steps to encourage immediate purchase.

69. C - The steps in the AIDA model are awareness, interest, desire, and action

69. Rodi owns Hallman's auto repair service. He has observed over the years that customers keep their high-mileage cars longer when the economy is doing poorly, creating demand for his maintenance and repair service. Rodi has observed the impact of _______ on demand for his service. A. break-even points B. the price inelasticity ratio C. the income effect D. the target return effect E. cross-price elasticity

69. C- This is an example of the income effect, where demand tends to shift from lower-priced to higher-priced products as consumers' income increases, and vice versa. In this case, keeping one's car and having it repaired costs less than getting a new car

70. Frederica manages an upscale women's clothing store. She wants more information about her customers' general feelings about upcoming fall fashions. Frederica will most likely use __________ to gather this type of data. A. door-to-door surveys B. focus group interviews C. syndicated data D. sales invoices E. census data

70. B - Focus groups are a good way to collect the feelings or opinions of a group of individuals when you do not need in-depth responses from each individual

70. Another name for a manufacturer's brand is a(n) __________ brand. A. systematic B. extended C. national D. premium E. private-label

70. C - The terms manufacturer's brand and national brand both refer to brands owned and managed by manufacturers.

70. After Sharon graduated from college, she found a steady and good-paying job, got married and began to raise a family, and began to receive recognition at work and in the community. Eventually, she began to devote more time and effort to intellectual and aesthetic pursuits. Sharon is moving up A. the conventional morality life cycle. B. the lifestyle achievement matrix. C. the Consumer Validation Model. D. Maslow's hierarchy of needs. E. from an internal to an external locus of control.

70. D - Sharon's steady job took care of the two lowest levels in Maslow's hierarchy (physiological and safety needs). Marriage took care of the third level (love), and recognition addressed needs for esteem, the fourth level. Sharon's current activities address the final level, self-actualization

70. Karen has identified four potential market segments for her Rent-A-Chef home meal catering service. The next step is to A. categorize each market segment by consumer demographics. B. establish her overall objectives. C. decide on a targeting strategy. D. develop a positioning strategy. E. evaluate the attractiveness of each segment.

70. E - After identifying potential market segments, the next step is to evaluate the attractiveness of each segment.

70. In a vertical marketing system, if the system is _______, the less likely conflict will occur A. less formal B. more independent C. less independent D. more conventional E. more formal

70. E - In a vertical marketing system, the members act as a unified system; the more formal the vertical marketing system, the less likely conflict is to ensue.

71. In an administered vertical marketing system, A. no individual participant has control over the others, since a third-party administrator oversees the entire supply chain B. independent firms at different levels of the marketing channel join through contracts to obtain economies of scale and coordination to reduce conflict C. there is no common ownership, and the dominant member has significant power to impose its ideas and objectives. D. independent firms join together formally to decide as a group how the marketing channel will operate E. participants—such as warehouses, transportation companies, and retail outlets—are typically owned by a parent company to ensure harmonious relations throughout the supply chain.

71. C - A powerful firm can manage a vertical supply chain through its power; this is an administered vertical marketing system.

71. The primary ethical dilemma facing managers is how to balance A. profits and expenses. B. employees' needs with the needs of the firm. C. shareholder interests with the needs of society. D. costs and benefits. E. ethical actions with making money.

71. C - Although firms cannot stay in business without earning a profit, using profit as the sole guiding light for corporate action can lead to short-term decisions that cause the firm to lose customers in the long run. The primary ethical dilemma facing managers is how to balance shareholder interests with the needs of society.

71. Karen has identified four potential market segments for her Rent-A-Chef home meal catering service. She will now evaluate whether or not each segment is attractive relative to all of the following except A. substantial B. responsive. C. profitable. D. reachable. E. realistic.

71. E - When evaluating a segment's attractiveness, we consider whether the segment is substantial, responsive, profitable, reachable, and identifiable

72. The basic reason manufacturers spend time and money building their own brands is to A. create brand awareness. B. build brand equity. C. offset the power of private-label manufacturers. D. create positioning possibilities for their generic product lines. E. maximize product line depth.

72. B - Companies invest in brands to build brand equity, which is a corporate asset

72. Franchising involves a(n) __________ vertical marketing system A. cooperative B. corporate C. contractual D. administered E. conventional

72. C - Franchising is an example of a vertical marketing system that is coordinated through contractual relationships. In fact, franchising is the most common type of contractual vertical marketing system

72. There are approximately 1 billion people living in India. Only about 200 million of these people earn more than the equivalent of $1,000 per year. According to Maslow's hierarchy, most of the other 800 million Indian consumers are primarily addressing their __________ needs. A. social B. personal C. psychological D. esteem E. physiological

72. E - Many people in India are living at a subsistence level, able to afford little but basic necessities (if that). This indicates that they are at the lowest level in Maslow's hierarchy, physiological needs.

72.In the selling process, the saying "It ain't over till it's over," refers to the _______ stage of the process. A. generating and qualifying leads B. preapproach C. sales presentation D. closing the sale E. follow-up

72. E - This refers to the follow-up stage. It is essential to perform well in this stage to build relationships and earn additional orders in the future

73. Procter & Gamble is a huge national brand manufacturer. By owning its brands like Tide and Crest, P&G A. can monopolize store brands. B. has greater opportunity to dictate retail pricing. C. has greater control over its marketing strategy. D. can increase brand dilution. E. can eliminate any local competition.

73. C - Brand ownership gives the owner greater control over its marketing strategy—the ability to position the brand and to establish the marketing mix

73. Tyree is concerned about a competitor's new line of outdoor barbeque grills that provide most of the same features as his products. Tyree might consider using __________ to gather qualitative information from consumers about the competitor's offerings. A. industry surveys B. experiments C. focus groups D. observation E. primary data mining

73. C - Focus groups are a good way to collect the feelings or opinions of a group of individuals when you do not need in-depth responses from each individual

74. Felicia had just taken over her family's business after spending ten years in the marketing department of a large corporation. She met with a representative from one of her firm's biggest customers, who told her, "We should think about how we can make the pie bigger rather than fighting over the size of the slices." She had expected a more cutthroat approach rather than this call for a A. partnering relationship B. shared mission statement C. common marketing system D. corporate vertical marketing system E. linked supply chain

74. A - Felicia's customer is proposing a partnering relationship, in which she and the customer will look for opportunities that are mutually beneficial and will seek to maintain the relationship over the long term

74. Cross-price elasticity is the A. percentage change in quantity of a product demanded divided by the percentage change in its price. B. percentage change in quantity demanded of product A compared to the percentage change in price of product B. C. change in price of product A divided by change in quantity demanded for product B. D. change in quantity of a product demanded divided by the change in its price. E. change in quantity of a product demanded divided by the change in its elasticity.

74. B - Cross-price elasticity looks at the impact of a change in one product's price on demand for another product. The two products might be substitutes or complementary products

74. During the _____ phase of the strategic marketing planning process, ethical mission statements can serve as a means to guide a firm's SWOT analysis. A. brainstorming B. planning C. implementation D. control E. revision

74. B - It is during the planning phase that ethical mission statements would serve as a means to guide a firm's SWOT analysis.

74. Brands that are owned by ___________ are called private-label brands. A. manufacturers B. wholesalers C. supply chain specialists D. retailers E. manufacturer's reps

74. D - Brands owned by retailers are called store brands, house brands, own brands, or private-label brands

74. Though he has never owned a Jaguar, Jerry thinks they are poorly made and have many mechanical problems. For Jaguar to sell Jerry a car, the company would need to change the __________ component of Jerry's attitude. A. social B. affective C. functional D. cognitive E. physiological

74. D - These are beliefs, which are part of the cognitive component of Jerry's attitude. The affective component is represented by his emotional responses to the car (he may feel anger, remembering a time he rode in someone else's Jaguar and it broke down). The behavioral component is represented by what he actually does (driving past a Jaguar dealer and not even looking at the cars as he goes by)

75. In the infamous Coke/New Coke taste test, 54 percent of consumers, using a blind taste test, preferred the New Coke formula to the existing formulation. This is an example of a(n) __________ marketing research method. A. qualitative B. quantitative C. observational D. syndicated E. invalid

75. B - Quantitative research is numeric/statistical in nature, as in this case where the precise percentage of consumers preferring one option to the other was measured. Formal studies such as specific experiments, scanner and panel data, or some combination of these are quantitative in nature

75. Apple computer users tend to like the company and love its products. Apple has nurtured this __________ component of its customers' attitudes. A. social B. affective C. psychological D. cognitive E. physiological

75. B - The words "like" and "love" are important here because they represent feelings, which make up the affective component of an attitude. The cognitive component would be represented by beliefs (for example, the belief that Apple is better at innovation than its competitors). The behavioral component would be represented by actions (for example, the choice to buy an Apple computer instead of a Windows PC).

75. Barbara is using a test model of her firm's new inline skates to see if they work according to the design specifications. Barbara is involved in A. test marketing. B. product launch. C. alpha testing. D. target marketing. E. concept testing.

75. C - In alpha testing, the firm attempts to determine whether the product will perform according to its design and whether it satisfies the need for which it was intended

76. When comparing the various communication channels available to marketing professionals, it becomes apparent that A. consumers prefer advertising over other channels. B. no single channel is better than another channel. C. public relations is the least expensive, but the most successful channel. D. personal selling is the most expensive and the least successful channel. E. online marketing is taking the place of advertising and public relations

76. B - No single channel is necessarily better than another channel; the goal of IMC is to use them in conjunction so that the sum exceeds the total of the individual channels.

76. __________ refer(s) to the process by which consumers select, organize, and interpret information. A. Attitude B. Learning C. Perception D. Values E. Consumption

76. C - This is the definition of perception

77. An advertisement for Titleist golf balls that airs on the cable television network Golf Channel is an example of which type of media channel? A. niche B. mass C. select D. product placement E. institutional

77. A - Niche media channels are more focused and generally used to reach narrower segments, often with unique demographic characteristics or interests

77.Sally has been having a difficult time working with a particular buyer while using the personal selling process, and she has asked her manager, Chris, for some ideas about how to close the sale effectively. Chris asks her a number of questions to help sort out the difficulties. Which of the following questions would Chris be least likely to ask? A. "What does the buyer's body language tell you about her readiness to buy?" B. "Did you skip any steps in the process?" C. "Are there any reservations that have not been addressed to the buyer's satisfaction?" D. "Have you completed the follow-up to ease the buyer's mind?" E. "Should you go back to an earlier stage in the process and start the process over from there?"

77. D - Follow-up happens after closing the sale, so this is not a question Chris would ask of Sally at this stage

78. Learning refers to a change in a person's thought process or behavior that arises from A. attitudes. B. experience. C. risk analysis. D. cultural norms. E. postpurchase dissonance.

78. B - Learning arises from experience, through which new information is introduced

78. What is the problem associated with service quality standards such as "be nice" or "do what the customers want"? A. They create low expectations B. They are not specific. C. They do not allow for the voice-of-customer process. D. Most employees are unwilling to do what customers want. E. They create a delivery gap.

78. B - Service providers cannot meet the firm's standards if provided with only general instructions (as in "do a good job"). Specific standards must be provided

78. College students can be a less __________ market segment because students' media habits are quite diverse and firms might have to use a wide variety of media to attract this segment. A. substantial B. perceptive C. identifiable D. reachable E. quantifiable

78. D - Although there are many potential ways to reach college students, their media habits diverge so much that it can be difficult to reach them without using a wide variety of media.

78.Which of the following is the best example of a mass media advertising channel? A. Weight Watchers Magazine B. Red State Talk Radio (WRS) C. Home and Garden TV (HGTV) D. USA Today E. The Golf Channel

78. D - Mass media advertising channels are ideal for reaching large numbers of anonymous audience members, and USA Todayreaches the broadest national audience

78. All of the following are interactive elements of an IMC strategy except A. personal selling. B. consumer contests. C. mobile marketing. D. public relations. E. online marketing.

78. D - Personal selling, consumer contests, mobile marketing, and online marketing all involve direct interaction with a customer, as well as customer participation. Public relations involves building and maintaining a positive image, and customers do not need to take any action to receive it, making it indirect.

79. The proliferation of new media alternatives has led many firms to shift their promotional budgets from A. publicity to public relations. B. personal selling to advertising. C. integrated marketing communications to disintegrated marketing communications. D. indirect marketing to viral marketing. E. advertising to direct marketing and website development.

79. E - Marketing dollars have recently been shifted toward direct marketing, website development, and product placement.

80. A(n) __________ is a supply chain whose members act like a unified system A. vertical marketing system B. independent marketing system C. concentrated marketing system D. conventional marketing system E. strategic marketing system

80. A - A vertical marketing sysxf115tem is a supply chain in which the members act as a unified system and there are three types: administered, contractual, and corporate

80. Lorraine belongs to a national consumer panel created by a market research company. She regularly receives samples of new products from a variety of firms and fills out questionnaires about the products. The national consumer panel Lorraine is part of is engaged in A. premarket testing. B. product launch. C. test marketing. D. product development. E. concept testing.

80. A - Panels of this type are often used for premarket testing, which often combine product sampling with surveys collecting consumers' reactions to the product.

80.__________ is a particularly good advertising medium for groceries and fast food. A. Television B. Internet C. Direct mail D. Radio E. Newspaper

80. D - Radio is a good medium for products such as grocery purchases or fast food because many consumers decide what to purchase either on the way to the store or while in the store. Because many people listen to the radio in their cars, it becomes a highly effective means to reach consumers at a crucial point in their decision process

81._______ refers to the process of evaluating and selecting the _______. A. Advertising; media buy B. Media planning; media mix C. Promotion planning; advertising mix D. Media buying; media mix E. Media mixing; media buy

81. B - Media planning refers to the process of evaluating and selecting the media mix—the combination of the media used and the frequency of advertising in each medium—that will deliver a clear, consistent, compelling message to the intended audience.

81. Christopher bought Timberland boots because he felt they were perfect for his outdoor activities. Patrick bought the same kind of boots because he felt they were stylish, especially with the logo clearly visible. The psychological factor driving Patrick's behavior is A. a functional need. B. lifestyle. C. brand association. D. learned behavior. E. his evoked set.

81. B - Of the factors listed, lifestyle is the best choice because it represents the way Patrick chooses to spend his time and money. He wants to be stylish, and so chooses the boots. This is an example of a psychological, not a functional need

81. Sales representatives are often compensated, at least in part, on a percentage of the sales revenue. This percentage is known as a A. bonus. B. sales increment. C. base salary. D. commission. E. finder's fee.

81. D - This percentage is a commission. It is common for part or all of a sales rep's compensation to be based on commission.

82. A(n) __________ gap exists when a firm knows what it needs to do to meet customers' service expectations, but sometimes fails to do it. A. knowledge B. communication C. standards D. empowerment E. delivery

82. E - A delivery gap is a gap between service standards and actual delivered service.

82. Caroline is assessing market growth, market competitiveness, and market access for each segment she has identified. Caroline is assessing the __________ of each potential market segment. A. substance B. responsiveness C. identification D. reach E. profitability

82. E - Evaluation of profitability considers market growth, market competitiveness, and market access, along with ensuring that the profit margin will be sufficient to cover costs

84. After assessing the market growth potential and market competitiveness for his company's baby products in Mexico, Harmon wanted to evaluate market access. To do this, Harmon would consider A. ease of accessing or developing distribution channels and brand familiarity. B. the current size of the market and the expected growth rate. C. ease of pricing control and number of promotional outlets. D. the number of competitors, entry barriers, and product substitutes. E. profitability and customer buying behavior.

84. A - Evaluation of profitability considers market growth, market competitiveness, and market access, along with ensuring that the profit margin will be sufficient to cover costs. Market access includes ease of developing or accessing distribution channels, and brand familiarity

84. In questionnaire design, a question such as "Do anti-lock braking systems reduce car accidents?" A. is a question that respondents cannot easily or accurately answer. B. is a question that respondents are reluctant to answer because the information is sensitive. C. steers respondents to a particular response. D. asks two questions at once. E. is one sided, presenting only one side of an issue.

84. A - Respondents are unlikely to have sufficient information to answer the question.

84. Empowerment is when __________ are authorized to make decisions to help their customers and, as a result, service quality generally improves. A. consultants B. middle managers C. corporate executives D. production control managers E. frontline employees

84. E - Empowerment refers to permitting frontline employees to make decisions about service delivery to meet customers' specific needs. In general, empowerment of employees results in better service

84. The best way for sales managers to instill ethical behavior in the sales force is to A. provide extensive ethical training. B. use only straight salary and no commissions. C. employ a sales ethics forecaster. D. never use telemarketing. E. lead by example.

84. E - Sales managers need to lead by example in order to encourage their employees to behave ethically

85. Which of the following would not be used in calculating the profitability of a segment? A. segment perceptions B. segment size C. segment adoption percentage D. profit margin percentage E. fixed costs

85. A - A calculation of segment profitability uses segment size, estimated segment adoption percentage, purchase behavior (frequency and price), profit margin percentage, and fixed costs

85. Many Hollywood movie stars were among the first to buy electric and hybrid vehicles. These stars often became __________, influencing other consumers' behavior. A. reference groups B. internal loci of control C. cultural icons D. cognitive parameters E. evoked images

85. A - Celebrities can serve as a reference group, since some consumers like to identify with celebrities and to follow their lead

85. Using technology and __________ are two ways a delivery gap can be reduced in size. A. empowering employees B. variability analysis C. voice-of-customer analysis D. zone of tolerance analysis E. public relations

85. A - Empowerment refers to permitting frontline employees to make decisions about service delivery to meet customers' specific needs. In general, empowerment of employees results in better service.

86. A _______ refers to the use of the same brand in a different product line. A. line extension B. brand extension C. product mix extension D. co-brand E. brand license

86. B - This is the definition of brand extension

87. The most common type of contractual vertical marketing system is a A. wholesale agreement B. joint venture C. licensing arrangement D. partnership E. franchise

87. E - Franchising is the most common type of contractual vertical marketing system

88. In recent years, the component of IMC that has received the greatest increase in aggregate spending is A. media advertising. B. direct marketing. C. public relations. D. sales promotions. E. publicity.

88. B - Direct marketing has received the greatest recent increase in aggregate spending

88. Most beverage distributors have their delivery people act as the firm's sales representatives. The delivery people primarily function as A. order getters. B. order takers. C. sales support personnel. D. manufacturer's reps. E. telemarketers.

88. B - The delivery people will be primarily calling on existing customers, processing routine orders for beverages. This is what order takers do

88. Ronaldo wants to develop an Internet auction-based business and is working through the STP process. After establishing his objectives, describing potential market segments, and evaluating the attractiveness of each segment, Ronaldo now has to A. differentiate his product line. B. consider future customer loyalty. C. create a perceptual map. D. select a target market. E. divide geographic segments into lifestyle groups.

88. D - After setting objectives, forming segments, and evaluating segment attractiveness, the next step of the STP process is to select one or more target markets

88. When supply chain members view their goals and ultimate success as intricately linked, or ________, they develop deeper long-term relationships A. conventional B. vertical C. horizontal D. administered E. interdependent

88. E - Interdependence between supply chain members that is based on mutual benefits is a key factor in developing and sustaining the relationship.

89. In a retail supply chain, the ________ records the purchase information and electronically sends it to the corporate office A. UPC B. POS terminal C. VMI D. ASN E. data warehouse

89. B - The point-of-sale (POS) terminal records the purchase information and electronically sends it to the corporate office.

89. Sue spends much of her time checking inventories, processing straight rebuys, and making sure that everything is going smoothly. Sue is primarily a(n) A. business development specialist. B. caretaker rep. C. order getter. D. order taker. E. sales support rep.

89. D - Sue is handling routine orders and checking inventory, which are tasks common to order takers

89. Brenda wants to reduce mass media IMC and to increase the use of personalized marketing communication messages. To achieve this goal, Brenda will likely increase her use of A. sales promotions. B. advertising. C. public relations. D. price reductions. E. direct marketing.

89. E - Direct marketing is the form of marketing communication that is most easily personalized.

90. the contribution per unit is A. price minus total costs B. price minus total variable cost C. price minus variable cost per unit D. total revenue minus total cost E. break-even quantity divided by total fixed costs

90. C - Contribution per unit = Revenue - Variable cost. This represents the revenue that is "left over" after covering variable costs

91. After observing a customer verbally abuse a server, the first thing a manager can do to ensure quality service is to A. throw the customer out of the restaurant. B. assume the server provoked the attack and respond accordingly. C. provide emotional support to the server. D. review the delivery support system. E. make sure services delivery expectations are consistent and coherent throughout the organization.

91. C - One reason service jobs can be difficult is that service employees must remain pleasant and courteous even when customers are not. Managers can help frontline employees manage angry customers with courtesy by providing emotional support and incentives

92. Managers of fast-food restaurants struggle with a rapid turnover of personnel. Employee turnover rates of 100 to 200 percent annually are common. The work environment is difficult and customers can often be demanding. One of the first steps managers can take to help workers deliver quality service is to A. ban abusive customers from their restaurants. B. reward service providers based solely on the speed of service. C. provide emotional support and concern for their employees. D. review the delivery support system. E. make sure services delivery expectations are consistent and coherent throughout the organization.

92. C - One reason service jobs can be difficult is that service employees must remain pleasant and courteous even when customers are not. Managers can help frontline employees manage angry customers with courtesy by providing emotional support and incentives

92. Regina wants to position her financial services company. Regina can position her services according to all of the following except A. the value proposition. B. product attributes. C. symbols. D. competitive comparisons. E. profitability.

92. E - Each of these is an option to consider when positioning a product except profitability.

93. Whenever the president of the local public university promotes the institution, he emphasizes the university's price (much lower than neighboring private colleges) and high quality. He is positioning the institution based primarily on A. the value proposition. B. product attributes. C. symbols. D. competitive comparisons. E. profitability.

93. A - By emphasizing the low price and high quality, the university president is focusing on value

93. If a marketing researcher has to collect data under severe time constraints, which of the following types of data sources would probably be available soonest? A. syndicated data B. focus group data C. experimental data D. panel data E. survey data

93. A - Syndicated data can be purchased and are readily available for a fee

93. __________ represents the systems and equipment resources that service providers need to be able to close the delivery gap. A. Service infrastructure B. Quality mechanics C. Instrumental support D. Dynamic support E. Customer interface architecture

93. C - This is the definition of instrumental support—the systems, equipment, and tools frontline employees need to do their jobs

93. The growth phase of the product life cycle is always dynamic. Which of the following does not occur in this phase? A. Profits increase as sales increase. B. Profits increase as economies of scale are attained. C. Some new competitors may enter the market with similar products. D. Some competitors will exit in an "industry shakeout." E. Price competition with protracted price wars erodes profits.

93. E - Price competition and price wars are more likely to emerge in the maturity stage

94. Which of the following is not one of the important functions of labels on products and packages? A. protecting against damage to the product B. providing consumer information to assist in purchasing C. identifying the brand and building brand imag D. promoting the brand or complementary brands E. listing the ingredients

94. A - The package might protect against damage to the product, but that is not a function of the label.

94. Brad is working on the creative design for his company's new ad campaign. This is his first major assignment. His boss reminds him that when writing a headline, it should A. represent the main text of the ad. B. identify the sponsor of the ad. C. use jargon and complex terminology. D. overshadow the visual message. E. be short and use simple words.

94. E - The headline is the large type in an ad that is designed to draw attention. It should be short and use simple words.

95. The number of participants who discontinue their use of a service divided by the average number of total participants is called A. churn. B. customer retention. C. customer lifetime value. D. cancellation cost. E.customer loyalty

95. A - Churn is the number of participants who discontinue their use of a service divided by the average number of total participants. It is a measure of how much turnover a company experiences in its customer base.

95. In a market with _______ there are many firms providing differentiated products. A. pure competition B. oligopolistic competition C. monopolistic competition D. a monopoly E. a duopoly

95. C - This is a monopolistic competition situation, where many firms compete and each seeks to differentiate its offerings

95. Many states have laws regulating the prices businesses charge during emergencies like hurricanes. These laws are designed to protect consumers whose __________ state may impair their ability to make sound purchase decisions. A. shopping B. social C. theoretical D. temporal E. physical

95. D - A temporal state is a temporary state of mind, such as the fear that would be aroused by an impending hurricane.

95. After installing a(n) _______ in its JIT system, Chocolate Tree (a retail chocolate store) was able to reduce lead time for merchandise orders A. supply chain B. distribution center C. quick change directory D. electronic data interchange system E. checking system

95. D - Lead time is the time it takes to receive merchandise from the point when a decision is made to place an order. EDI can reduce lead time by electronically transmitting information between the buyer and seller

96.The major text portion of a print advertisement that should arouse a desire for the product and move the consumer to action is contained in the A. headline. B. body copy. C. brand elements. D. photo credits. E. subheads.

96. B - The body copy represents the main text portion of the ad. It builds on interest generated by the visual and headlines, explains in more depth what the headline and subheads introduced, arouses desire for the product, and provides enough information to move the target consumer to action.

96. Because there are many firms in monopolistic competition markets, A. everyone is a price taker B. producers do not have to consider the reactions of rival firms C. government often encourages consolidation to reduce the number of competitors D. price controls may be implemented. E. the many competitors will focus on product differentiation.

96. E - Firms in monopolistic competition markets seek to differentiate their offerings in order to target unique segments

97. Which of the following best describes a company sales force? A. A company sales force is composed of people who are employees of the selling company. B. A company sales force is made up of independent agents making cold calls. C. A company sales force is a team set up to sell products to companies. D. A company sales force is an organization that supplies sales reps to other companies E. A company sales force is composed of people both inside and outside the organization.

97. A - A company sales force is composed of people who are employees of the selling company

98. While JIT systems have many benefits, they A. make the logistics function more complicated B. interfere with new product development functions C. confuse customers D. work only in vertical marketing systems. E. decrease the accuracy of demand forecasts

98. A - JIT systems, despite their benefits, increase complexity and transportation costs associated with distribution. It is simpler and more straightforward to place a small number of large-quantity orders and then to stockpile inventory until it is needed; however, for many firms the benefits of JIT far outweigh the cost of the greater complexity

98. Which of the following is most likely to be characterized by pure competition in the United States? A. soy beans B. cereal C. soft drinks D. computer operating systems E. fast-food restaurants

98. A - Pure competition refers to a market in which firms provide similar, undifferentiated products. This includes commodities like soybeans, salt, and sugar

98. A restaurant chain is working on improving the quality of its food and service. To track its progress it recruits customers who agree to respond to customer satisfaction surveys once every three months over the next two years. What kind of data is the restaurant chain collecting? A. panel data B. internal secondary data C. focus group data D. external secondary data E. scanner data

98. A - This is an example of using a panel—a group of consumers who provide information over a period of time. These are primary, not secondary, data because the data are being collected specifically to measure progress in improving satisfaction.

98. The __________ gap can be reduced by managing consumers' expectations and promising only what can be delivered or possibly even a little less. A. knowledge B. communication C. delivery D. standards E. empowerment

98. B - A communication gap is the difference between promised and actua Al service. By making realistic promises, the firm can manage expectations

98. The goals of IMC need to A. expand as the advertising budget expands. B. be explicit and measurable. C. be encoded before they can be decoded. D. include both the noise effect and the transmitter effect. E. be separate from budget.

98. B - All IMC goals should be explicitly defined and measured so that success can be evaluated

98. The CEO of David's firm just came back from a business seminar. He called David into his office and stated, "I just found out about the product life cycle curve in a strategy seminar. This is a great tool. I want you to drop whatever you're doing and inventory all of our products to determine exactly where they are in the product life cycle." David needs to think fast for a good answer because he knows that A. the product life cycle is not a useful concept. B. it is often impossible to identify with precision where a product is in the product life cycle. C. the product life cycle is not helpful in companies like his that pay attention to diffusion of innovation. D. the idea has been discredited by recent research. E. the product life cycle works only for certain types of firms.

98. B - Although the product life cycle concept can be useful in guiding marketing strategy, it is difficult and often impossible to determine precisely where a product is in the product life cycle

99. Because there are many firms with similar products in purely competitive markets, A. price is determined by the laws of supply and demand. B. consumers develop personal preferences. C. firms find it easy to build strong, distinct brands. D. advertising is heavily used. E. the many competitors will focus on variable cost pricing.

99. A - Since products in purely competitive markets are not differentiated, price tends to fluctuate based on supply and demand

99. Within a perceptual map, a(n) __________ represents where a particular market segment's desired product would lie. A. point of parity B. strategic target C. PRIZM cluster D. ideal point E. target position

99. D - The ideal point is the point representing the target market's "perfect" product offering.

99. Physicians regularly overstate the expected recovery time from surgery, knowing that managing patients' expectations will reduce the __________ gap associated with their service. A. knowledge B. empowerment C. delivery D. standards E. communication

99. E - A communication gap is the difference between promised and actual service. By making pessimistic predictions for recovery time, doctors can manage patients' expectations

One example of a customer loyalty program is A. a "frequent diner" card at a restaurant, offering a free appetizer for every $100 in food purchases. B. a quantity discount offered for large purchases at an office supply store. C. seasonal sales on top-selling items. D. an "everyday low price" policy on all products at a grocery store. E. an extensive customer service training program for new employees at a hair salon.

A - A "frequent diner" program gives a customer an incentive to visit the restaurant more often, even if competitors try to win customers away with special menus or other offers, because the customer wants to earn the rewards that come with frequent visits

30. Manufacturers would prefer to produce in a country with a trade __________, because it signals a greater opportunity to export products to more markets. A. surplus B. deficit C. discrepancy D. bonus E. balance

A - A country with a trade surplus does more exporting than importing. This offers an advantage to manufacturers in that country

After identifying various market segments that her company could pursue, Lisa evaluated each segment's attractiveness based on size, income, and accessibility. Lisa was involved in A. target marketing. B. situation analysis. C. diversification. D. positioning. E. market penetration estimation.

A - A key component of target marketing is the evaluation of potential target segments' attractiveness.

105. For which of the following purchases would consumers most likely engage in limited problem solving? A. college courses B. a new car C. a hom D. coffee to go, for a regular coffee drinker E. a location for a large wedding reception

A - A new car, a home, and a wedding location are major decisions that will probably lead to extended problem solving. Coffee to go for a regular coffee drinker is probably a habitual purchase. But college courses will probably lead to limited problem solving. Students will check available options, consider their choices, and might consult friends or past course evaluations for more information; however, they are unlikely to engage in extensive information search or extensive evaluation of alternatives, especially since some courses will be required for a degree

Though Asian Americans comprise only about 6 percent of the U.S. population, they represent A. the fastest-growing minority population. B. the easiest minority group to access. C. a large proportion of the minorities in the Midwest. D. a uniform group of consumers with a common language and cultural background. E. all of these

A - Asian Americans are the fastest-growing ethnic minority in the United States.

98. Which of the following is a potential negative factor for foreign investment in China? A. China's population is aging rapidly. B. China drastically restricts the goods it allows U.S. companies to export to China. C. China's standard of living has dropped over the past 30 years. D. China has imported fewer goods from the United States each year for the past decade. E. Chinese consumers are not interested in purchasing products from the United States.

A - Because of China's population controls limiting families to one child, it has one of the most rapidly aging populations in the world, which may affect its attractiveness as a consumer market in the future

During a SWOT analysis, a company should assess the opportunities and uncertainties of the marketplace due to changes in several factors, given the acronym CDSTEP. Which of the following is not one of these factors? A. ethical B. cultural C. political D. technological E. demographic

A - CDSTEP stands for cultural, demographic, social, technological, economic, and political forces

Craig sees that his company's quarterly sales and profits are significantly above projections and says, "That's great. Let's keep doing what we've been doing." Craig is ignoring the __________ step of the marketing planning process. A. evaluate performance B. define the business mission C. perform situation analysis D. implement marketing mix and resources E. identify and evaluate opportunities

A - Craig should still evaluate performance, even if results were good. There's always a chance that he was successful due to luck, not due to the quality of his plan. There's also a chance that he did well, but that he could have done even better with a stronger plan

77. Global marketers typically find distribution in developing countries is more complex because A. they must go through many different types of distribution channels. B. distribution is more heavily regulated in developing countries. C. most consumers in developing countries live in densely populated cities. D. the infrastructure is more advanced in most developing countries. E. consumers in developing countries have very specific preferences.

A - Distribution channels tend to be longer and more complex within global contexts than in domestic markets. For this reason, distribution channel efficiency is a critical factor in keeping down costs in global marketing

91. Which of the following types of research would be considered quantitative research? A. experimental research B. observational research C. focus group research D. social media monitoring E. in-depth interviews

A - Experimental research manipulates a variable to establish cause-and-effect relationships. As a form of quantitative research, it is typically numeric/statistical in nature and can be used to confirm insights

86. When entering into a franchise agreement, what term is used to refer to the firm that is granted the right to operate a business using the franchise name and business concept? A. franchisee B. franchisor C. franchise agent D. franchise partner E. franchised owner

A - Franchising is a contractual agreement between a firm, the franchisor, and another firm or individual, the franchisee. The franchisee can operate a business using the name and business format developed and supported by the franchisor

34. Select the statement that best describes the key traits of Generation Y. A. This generation varies the most in age, ranging from teenagers to adults who have their own families. B. This generation is the largest population of 50-plus consumers. C. This generation is the group that was born immediately after World War II. D. This generation was the first generation of latchkey children. E. Members of this generation—also known as Digital Natives—have had access to the Internet for their entire lives.

A - Generation Y is the biggest cohort since the original postwar World War II boom. It also varies the most in age, ranging from teenagers to adults who have their own families

64. Fernando was thrilled to find out that his company had just decided to invest a great deal of money in the product he was managing. He knows that even with its recent high rate of growth and the fact that it dominates its market, he would need more money to establish it firmly. Using the BCG portfolio analysis, his product would be classified as a(n) A. star. B. cash cow. C. question mark. D. dog. E. anchor.

A - In BCG analysis, a product with high relative market share in a high-growth market is called a star. Stars typically require investment in order to continue to grow and to maintain or improve their market positions

38. Cory is working on a global marketing assessment team looking out well into the future to help determine the most attractive market areas around the world. He is evaluating market sizes and growth rates. Based on population growth rates in different regions, he should consider that A. countries with high purchasing power today may not continue to show the same growth in the future. B. the United States and Western Europe will have dramatic increases in population growth leading to overcrowding. C. the middle class in India will continue to shrink as the rich get richer and the poor get poorer. D. in places like India, urban population centers will become increasingly unattractive and the rural areas will experience major growth in population. E. the global population is expected to grow at staggering rates indefinitely.

A - In general, developed countries are experiencing zero or no population growth, while most less developed nations are experiencing rapid population growth. These trends are expected to shift today's patterns of purchasing power. The countries with the highest purchasing power today may become less attractive in the future for many products and services because of stagnated growth

When pursuing a market development strategy, expanding into international markets is generally A. riskier than expansion in domestic markets. B. impossible due to negative attitudes about American products. C. executed with the help of international trade subsidies. D. simplified by creating new products for export markets. E. the only option offering substantial opportunities for growth.

A - International expansion is usually riskier because there are many differences—cultural differences, economic considerations, and political and legal differences, for example—to consider

58. Generally, firms entering foreign markets begin with A. less risky strategies first. B. direct investment. C. importing. D. decentralized production. E. the riskiest, but most profitable endeavor.

A - Many firms choose to enter foreign markets first with less risky strategies (such as exporting), moving to riskier strategies as their knowledge and experience grows

In marketing terms, the trade of things of value between the buyer and the seller so that each is better off as a result is referred to as A. a marketing exchange. B. value cocreation. C. the marketing mix. D. a value transaction. E. relational marketing.

A - Marketing is fundamentally about an exchange—the trade of things of value between the buyer and the seller—so that each is better off as a result

111. Franco, a former retailer, has been living in the United States for five years and wants to start a business. He does not have an existing firm or a product, and he doesn't have a lot of capital, but since he loves McDonald's food, he decides to A. open a McDonald's franchise. B. directly invest in McDonald's. C. export McDonald's products to other countries. D. form a strategic alliance with McDonald's. E. form a joint venture with McDonald's.

A - McDonald's is a global franchisor. A franchising contract allows the franchisee to operate a business—a retail product or service firm or a B2B provider—using the name and business format developed and supported by the franchisor

36. Economic measures like GDP and GNI do not fully account for a country's economic health because they measure only A. material output. B. international trade. C. global expectations. D. purchasing power parity. E. poverty potential.

A - The GDP and GNI look at material output of the economy, failing to take other issues (such as quality of life) into account

The results of the 2010 census suggest what about the 42 million African American U.S. households? A. They are more affluent and suburban. B. They are younger and less affluent. C. They are less affluent, but more suburban. D. They are older and less affluent. E. They are older and more urban.

A - The census of 2010 counts 42 million African American U.S. households, who are more affluent and suburban than previous studies suggested. They also tend to be younger

Laws that prohibit the formation of monopolies or alliances that would damage a competitive marketplace benefit consumers through A. increased choices. B. higher prices. C. protection from false advertising. D. fair debt collection practices. E. fewer competitors.

A - The concern about monopolies and alliances is that they will inhibit competition, possibly raising prices and reducing choice

27. The components of a global market assessment include all of the following except A. ethnic analysis. B. infrastructure and technological analysis. C. analysis of government actions. D. sociocultural analysis. E. economic analysis.

A - The four components of a global market assessment are economic analysis, sociocultural analysis, infrastructure and technological analysis, and an analysis of government actions or inactions

The political/regulatory environment comprises political parties, governmental organizations, and A. legislation and laws. B. citizens. C. interest groups. D. for profit and nonprofit businesses. E. international influences.

A - The political/regulatory environment comprises political parties, government organizations, and legislation and laws.

Many companies use the Internet to allow consumers to design customized products; for example, Nike allows customers to order shoes with custom color combinations. This is a form of A. micromarketing B. concentrated targeting. C. psychographic segmentation. D. differentiated targeting. E. undifferentiated targeting.

A - This is a form of micromarketing—on a one-to-one scale—because each consumer receives a customized product.

________ is the cost to the customers or the fee the bank charges those customers for borrowing money. A. Interest B. A service charge C. A tax D. A tariff E. A user fee

A - This is the definition of interest.

Yuri is considering a new promotional campaign in which he will compare his products to those of his competitors. Before initiating the promotional campaign, Yuri will assess his competitors' strengths, weaknesses, and A. likely reaction to his promotional activities. B. demographics. C. just-in-time processes. D. satisfaction quotient as perceived by customers. E. ethical values.

A - Yuri must consider competitors' strengths and weaknesses in order to perform accurate comparisons; he must also understand how competitors will respond to the promotional campaign

"Corporate pilot fish" are former employees who start a new business based primarily on contacts and contracts with their old company. A corporate pilot fish would probably pursue a(n) __________ targeting strategy. A. macromarketing B. micromarketing C. benefit-based D. differentiated E. undifferentiated

B - A corporate pilot fish will probably start with a very small group of customers and contacts, leading to a micromarketing strategy in which very small segments are targeted

28. Chris is gathering information about the general economic environment in Nepal. In doing so, he will look for information about the general economic environment, market size and population growth rate, and A. culture. B. real income. C. airport capabilities. D. political status. E. religious institutions.

B - A firm conducting an economic analysis of a country market must look at three major economic factors, using well-established metrics: the general economic environment, the market size and population growth rate, and real income

After defining the business mission, what should a firm do next to develop a marketing plan? A. conduct an STP analysis B. perform a situation analysis C. develop a positioning strategy D. select a target market E. implement the four Ps

B - A situation analysis is the second step of the marketing planning process

115. According to Hofstede's cultural dimensions concept, which BRIC country posts notably high scores in the dimensions of uncertainty avoidance and power distance? A. Brazil B. Russia C. India D. China E. Iran

B - As a country, Russia scores high on the dimensions of uncertainty avoidance and power distance. Although Hofstede was careful to warn that cultural dimension scores are informative only in a comparative sense, marketers clearly can use them to design strategies for the varied, promising, BRIC growth markets

Many states create licensing requirements for a variety of professionals (such as lawyers and accountants) designed to restrict entry into their market by professionals from other states. This strategy limits ____________ growth strategies. A. product proliferation B. market development C. market penetration D. diversification E. product development

B - By restricting the ability of lawyers and other professionals to easily expand their businesses across state lines, states are preventing these professionals from selling existing services to new customers. These would be market development opportunities

Country culture and regional culture are both part of a firm's A. demographics B. macroenvironment. C. political/legal environment. D. economic environment. E. immediate environment.

B - Country culture and regional culture are both part of a firm's macroenvironment

66. Of the five strategies for entering new markets, direct investment creates the A. least investment cost. B. greatest potential risk. C. most franchisee control. D. best opportunity for strong strategic alliances. E. greatest coordination of efforts of global and local partners.

B - Direct investment is the riskiest strategy for global expansion; however, it also offers the greatest potential rewards

When firms successfully implement poor strategies (perhaps due to good luck) or do a poor job of implementing good strategies, it can be difficult to A. generate cost-based performance ratios. B. evaluate performance and make adjustments. C. increase product excellence without reducing customer excellence. D. develop segmentation strategies. E. choose a business mission.

B - Good luck may mask the weaknesses in a bad strategy, and poor execution may mask the strengths of a good strategy, making evaluation difficult in both cases

31. The most common measure of market potential of an economy is a country's A. GNI. B. GDP. C. PPP. D. CPI. E. APR.

B - Gross domestic product (GDP) measures the market value of goods and services produced by a country in a year and is the most widely used of the various economic metrics

37. Today, many developed countries are experiencing __________ population growth. A. slight B. zero or negative C. rapid D. moderate E. significant

B - In general, developed countries are experiencing zero or negative population growth, while most less developed nations are experiencing rapid population growth

In most companies, portfolio management is typically done at the SBU or ___________ level of the firm. A. corporate B. product line C. customer care D. sales representative E. accounting

B - In general, marketing resources are allocated to SBUs or product lines. The corporate level is too high unless it is a very small company with just a single product line, and the sales representative level is generally too low

During which phase of the marketing planning process does a firm identify and evaluate different opportunities by engaging in segmentation, targeting, and positioning? A. planning phase B. implementation phase C. evaluation phase D. control phase E. strategy phase

B - In the implementation phase, marketing managers identify and evaluate different opportunities by engaging in segmentation, targeting, and positioning (STP) (Step 3)

109. Once a firm has done an analysis of the most viable markets for its products, then it must next A. determine the competition and develop strategies to overcome it. B. conduct an internal assessment of its capabilities. C. conduct an external analysis of the target market's economy, culture, and regulatory barriers. D. develop a product to meet the needs of those markets. E. achieve success with the product in its home market.

B - It must next conduct an internal analysis of its capabilities. This analysis includes an assessment of the firm's access to capital, the current markets it serves, its manufacturing capacity, its proprietary assets, and the commitment of its management to the proposed strategy.

72. Which of the following are the two components of a global marketing strategy? A. understanding foreign currency fluctuations and developing products that can be priced accordingly B. determining which target markets to pursue and developing a marketing mix to obtain a competitive advantage C. understanding the supply chain and distribution networks in foreign markets D. developing culturally appropriate advertising messages and cultivating "domestic" habits among foreign consumers E. adapting to foreign regulations and targeting as many people as possible

B - Just like any other marketing strategy, a global marketing strategy includes two components: determining the target markets to pursue and developing a marketing mix that will sustain a competitive advantage over time

__________ is an extreme form of a targeting strategy. A. Macromarketing B. Micromarketing C. Benefit marketing D. Differentiated targeting E. Concentrated targeting

B - Micromarketing is an extreme form of targeting, where a company caters to extremely small segments

After conducting STP analysis for her custom auto parts store and developing strategies for each of the four Ps, Monique now has to make _____________ decisions. A. competitive response B. resource allocation C. market growth D. product line E. mission statement

B - Monique has chosen a target market, determined positioning strategies, and developed marketing mixes. She now has to consider available resources and determine how they will be allocated across the different options available for implementation

45. In most cases, countries use tariffs to reduce foreign competition, but tariffs are also used A. to shorten supply chains. B. as a response to perceived unfair trade practices. C. to offer domestic discounts. D. to stimulate consumer demand. E. as a way to equalize quotas.

B - Tariffs are used primarily to raise the prices of imported goods; however, they can also be used to punish countries engaging in trade practices that are seen as unfair, by making it harder for their goods to find buyers

65. India, like some other countries, may require entering firms to create _________ when expanding into their markets, limiting outsiders' control of businesses. A. franchises B. export promotions C. joint ventures D. direct investments E. strategic alliances

C - A joint venture is a business formed when two or more firms pool their resources. It is a medium-risk strategy for global market entry. By requiring that foreign firms engage in joint ventures to enter the market, India helps ensure that a local firm can retain partial ownership of the venture

106. Which country's government has recently made significant changes that will modernize the retail environment, such as allowing joint ventures and direct ownership in some cases? A. Brazil B. Russia C. India D. China E. Mexico

C - Foreign retailers that carry multiple brands, like Walmart, are now allowed to own up to 51 percent of joint ventures in India, and retailers that carry only their own brand, like Nike, can now own 100 percent of their Indian businesses

Abercrombie & Fitch, a clothing retailer, includes a "SHARE" link on the product pages of its website. This link encourages an Abercrombie customer to post a link (perhaps showing a new style of jeans) on Facebook or Twitter. Abercrombie & Fitch hopes that the customer's friends (who are probably very much like current customers) will click the link, visit the page, and make purchases. This is an example of a __________ growth strategy. A. product proliferation B. market development C. market penetration D. diversification E. product development

C - In this situation, Abercrombie & Fitch is trying to sell more of its existing products to its current target market, which is a market penetration strategy. Market development would involve attracting new market segments, but the company is assuming that the friends of current customers will be similar to those customers (and thus part of the same market segment)

Which of the following statements accurately describes the current income distribution in the United States? A. The purchasing power of lower-income groups has been steadily rising. B. The middle class is outpacing all other income groups. C. Wealthy households are outpacing both poor and middle-class households. D. The income distribution among all households is becoming more equal. E. Wealthy households are declining rapidly in purchasing power.

C - Income distribution in the United States has grown more polarized—the highest-income groups are growing, whereas many middle- and lower-income groups' real purchasing power keeps declining

101. Which statement about India's population is true? A. With a median age of 61, India has one of the oldest populations in the world. B. India's young people mostly live in rural areas in large families. C. India's workforce is highly skilled, particularly in technology. D. India claims more than 25 percent of the world's population. E. Most Indian citizens shop in large retail outlets.

C - India's highly skilled workforce holds great attraction for firms that hope to expand using local talent, especially in technical fields

100. Why should marketers be aware of the BRIC countries? A. They are a microcosm of the rest of the world. B. They represent almost half the world's population. C. They are likely to be the source of most market growth. D. They have the most dramatic changes in culture and consumer buying patterns. E. They have stable population growth, which makes them easier to study.

C - The four BRIC nations are likely to be the source of most market growth

A marketing strategy identifies three things: ________, a related marketing mix, and the bases on which the firm plans to build a sustainable competitive advantage. A. customer value B. a mission statement C. marketing metrics D. a firm's target markets E. a product plan

D - A marketing strategy identifies three things: a firm's target market, a related marketing mix (its four Ps), and the bases on which the firm plans to build a sustainable competitive advantage

108. Which country has a rapidly aging population due to its one-child policy? A. Brazil B. Russia C. India D. China E. Japan

D - Although China's median age is slightly younger than that of the United States currently, at 36.3 years, the application of the one-child policy means that China is one of the most rapidly aging countries in the world.

80. Brands can be extremely valuable domestically, but challenging internationally. Companies can help overcome language difficulties in using brands by A. keeping the brand name the same in all languages, regardless of meanings, as long as the brand logo and symbol are displayed prominently. B. avoiding the use of the brand name in advertising and focusing on features and benefits. C. translating advertising copy for the entire ad except the brand name. D. developing brand names that have no preexisting meaning in any known language. E. adhering to the UN Convention on Naming Rights.

D - By choosing a meaningless brand name, the company is free to attach its own meanings to the name without the risk of the name suggesting unintended characteristics because of its meaning in a particular language.

In recent years, cellular (mobile) service providers have worked hard to eliminate "dead zones," providing customers with service wherever they travel. By working to make the network available in more locations, cellular service companies are focusing on __________ value creation. A. product B. price C. promotion D. place E. financial

D - By making sure the mobile network is available to use where and when customers want it, cellular service companies are working on place value creation

By offering environmentally responsible products, green marketers A. undercut prices of nonenvironmentally responsible marketers. B. keep costs much lower than those of competitors. C. make consumers feel guilty for buying other products. D. add value that other products do not have. E. all of these

D - Environmentally responsible products have additional value for consumers to whom protecting the environment is important.

63. Domestic firms developing a global entry strategy might consider franchising; however, the disadvantages need to be considered. Which of these is not a disadvantage of franchising? A. The franchisor has limited ability to ensure that foreign operations follow all the concepts and ideas that made the firm successful domestically. B. The franchisee might end up becoming a competitor. C. Franchising limits profit potential, since profits will have to be split with the franchisee. D. Franchising is the riskiest way to enter a foreign market. E. All of these are disadvantages a firm must consider.

D - Franchising is actually among the lower-risk global expansion strategies

Generational cohorts are groups of people of the same generation who have similar __________ because they have shared experiences and are in the same stage of life. A. regional cultures B. inflationary expectations C. political affiliations D. purchase behaviors E. cultural values

D - Generational cohorts are presumed to have similar purchase behaviors due to similar life experiences and similarity of life stage.

When marketers monitor the economic situation affecting their target markets, they are likely to monitor changes in all of the following except A. purchasing power. B. interest rates. C. inflation. D. age. E. currency exchange rates.

D - Income, interest rates, and inflation are among the economic factors; age is a demographic factor.

Sodexo is a corporation that manages school cafeterias, university dining halls, mess halls at military bases, concession stands at sports arenas, and other large-scale food service facilities. If Sodexo were to begin to sell individual frozen meals in supermarkets, it would be pursuing a __________ growth strategy. A. product development B. market development C. market penetration D. diversification E. product proliferation

D - Individually packaged frozen meals would be a new product area for Sodexo (even though it is still a food, the manufacturing process would be quite different from the bulk food service required by cafeterias and dining halls). Consumers in grocery stores would also be a new market, since Sodexo currently serves universities, the military, and the like. Thus, with a new market and a new product, this would be an example of diversification

107. Which country has embraced market-oriented economic development in spite of maintaining communist political ideals? A. Brazil B. Russia C. Iran D. China E. Afghanistan

D - Since 1978, China's leadership, while maintaining communist political ideals, has embraced market-oriented economic development, which has led to startlingly rapid gains

53. Chris laughed at some of the cultural mistakes companies made in advertising and promotion in international trade while he was in school. Now he was trying to determine what had gone wrong with the campaign he had planned in Latin America for his company's product, and it didn't seem quite as amusing. He narrowed the issues to sociocultural factors. He was looking at both __________ and __________. A. product uses; currency rates B. language; trading blocs and social structure C. potential tariffs; symbols D. visible artifacts; underlying values E. verbal communication; logistics

D - Sociocultural factors include visible artifacts like symbols and dress; they also include underlying values that can be more difficult to understand. Some of the other answers list one sociocultural factor (language, symbols, verbal communication) along with something that is not a sociocultural factor

Which statement about the changing ethnicity in the United States is true? A. Minorities now represent almost half of the population in the United States. B. In spite of increases in the Hispanic population, Hispanic buying power is expected to maintain its current level. C. Hispanics differ vastly from other groups in America in terms of consumer behavior. D. African American U.S. households are more affluent than previous studies suggested. E. Asian Americans are the slowest-growing minority population.

D - The 2010 U.S. Census counted 42 million African American U.S. households, who are more affluent and suburban than previous studies suggested

To determine how attractive a particular market is using the BCG portfolio analysis, __________ is(are) established as the vertical axis. A. competitive intensity B. sales dollars C. market size D. market growth rate E. market profit potential

D - The BCG matrix uses market growth rate on the vertical axis and relative market share on the horizontal axis. The other factors are not unimportant—they are simply not part of BCG portfolio analysis

In the immediate marketing environment, the first factor that affects the consumer is A. demographics. B. cultural values. C. social trends. D. the firm itself. E. technological advances.

D - The components of the immediate environment are the firm, its competitors, and its corporate partners.

Introducing newly developed products or services to a market segment the company is not currently serving is called A. product development. B. market development. C. market penetration. D. diversification. E. product proliferation.

D - This is the definition of diversification

Typical demographic data include all of the following except A. gender. B. income. C. race. D. education. E. language differences.

E - All of these are examples of demographic factors except language differences, which is a cultural trait

In mid-2010, Hewlett-Packard Company (HP) acquired Palm Computing Inc., a manufacturer of personal devices and smartphones. Before deciding to acquire the company, strategic planners at HP spent time thinking about how Palm and HP would "fit" together, and how the acquisition might change HP's core goals and objectives. The strategic planners were engaged in the ____________ step of the marketing planning process. A. perform situation analysis B. implement marketing mix and allocate resources C. identify and evaluate opportunities D. evaluate performance E. define the business mission

E - By considering questions of "fit" and changing objectives, the planners are considering HP's business mission and how the acquisition might change it.

114. If you visit a Kentucky Fried Chicken restaurant in China, along with KFC's regular menu items, you will find congee, a rice porridge that can feature pork, pickles, mushrooms, and preserved egg, on thewill find congee, a rice porridge that can feature pork, pickles, mushrooms, and preserved egg, on the menu. This is an example of which global product strategy? A. Sell the same products in both the home country market and the host country. B. Sell only products native to the home country. C. Sell a product similar to that sold in the home country, but include minor adaptations. D. Sell only products native to the various global markets. E. Sell totally new products or services.

E - Congee would be a totally new product

__________ segmentation is the most common basis of consumer market segmentation because these segments are easy to define and usually easy to reach. A. Geographic B. Psychographic C. Behavioral D. Benefits E. Demographic

E - Demographic segmentation tends to be easy to define because demographic factors are widely collected and so tend to be readily available.

90. When Ford Motor Company decided to sell the Fiesta—in the same form and design—around the globe, instead of selling different versions in different countries, this was part of Ford's global ________ strategy. A. communication B. pricing C. distribution D. exchange E. product

E - For the Fiesta, Ford has decided to go with the global product strategy of selling the same product in multiple countries

Gerald's Tire Store sets itself apart from competitors by the extra attention it pays to retaining loyal customers by providing fast, courteous service in a clean, professional-looking environment. Gerald's has used this strategy for over 40 years, expanding to 15 outlets. Gerald's Tire Store has created a ___________ macro strategy to create and deliver value and to develop a sustainable competitive advantage. A. supply chain excellence B. locational excellence C. product excellence D. diversification excellence E. customer excellence

E - Gerald's Tire Store has a macro strategy focusing primarily on customer excellence, based on the quality of customer service mentioned in the question

27. The centerpiece of the marketing environment analysis framework is A. the economy. B. corporate partners. C. culture. D. competitive intelligence. E. consumers.

E - The consumer is the center of all marketing activities

Successful firms focus their efforts on satisfying customer needs that A. are easiest to satisfy. B. provide minimal core value. C. are important to all generational cohorts. D. competitors have tried and failed to satisfy. E. match their core competencies.

E - The firm's competencies are its strengths; the firm is likely to have the most success if it focuses on these, finding customers whose needs it can meet using these strengths

The Mayo Clinic in Minnesota is known for top-quality medical care. For decades, even presidents and dictators from around the world flew to the Mayo Clinic to utilize its services. The Mayo Clinic used its reputation to create additional medical facilities in Jacksonville, Florida, and elsewhere. This is an example of a firm focusing its efforts on satisfying customer needs that A. are easiest to satisfy. B. provide minimal core value. C. are important to all generational cohorts. D. competitors have tried and failed to satisfy. E. match its core competencies.

E - The firm's competencies are its strengths; the firm is likely to have the most success if it focuses on these, finding customers whose needs it can meet using these strengths

Four frequently used targeting strategies are the micromarketing, undifferentiated, differentiated, and __________ targeting strategies. A. geographic B. benefit-based C. economic D. global E. concentrated

E - The four targeting strategies are micromarketing, undifferentiated, differentiated, and concentrated.

29. To determine the market potential for its particular product or service, a firm should use A. GDP data. B. unemployment data. C. purchasing power parity data. D. inflation data. E. as many metrics as it can obtain.

E - The market potential for a firm depends on many variables and no one metric can offer a complete picture of the economic environment in a country. As a result, the best approach is to use many measures in an analysis.

A product that is in a high-growth market but has a low market share would be classified as a ________ on the BCG matrix. A. ladder B. dog C. cash cow D. star E. question mark

E - This is the definition of a question mark. If the product is infused with resources, it has the potential to become a star

Briena is looking at the results of a syndicated study conducted two years ago. Briena is looking at primary data.

F - Data collected in advance of the current project is secondary data

The members of Generation X are also referred to as Digital Natives

F - Digital Natives are members of Generation Z, not Generation X

Distribution centers are always operated by distribution specialists

F - Distribution centers may be operated by retailers, manufacturers, or distribution specialists

An iPad mini advertisement appearing in the New York Times is an example of using a niche media channel

F - Niche media channels are more focused and generally used to reach narrower segments. Since the New York Times is geared toward reaching large audiences, it would be considered mass media

When survey data are analyzed in depth to determine consumer attitudes toward a product, this is an example of sentiment mining

F - Sentiment mining analyzes consumer comments on social media sites in order to gain insights about consumers' thoughts and preferences

As the number of communication media has increased, the task of understanding how best to reach target customers has become easier

F - The proliferation of communication media—driven largely by technology—has made it much more difficult to choose the best way to reach a target market.

One of the social trends discussed in the text that is shaping consumer values in the United States and around the world is a concern about the vast disparity in income

F - The social trends mentioned in the text are a greater emphasis on health and wellness concerns, greener consumers, and privacy concerns

Although innovation strategies may not work in the long run, overriding short-term reasons compel firms to introduce new products and services.

F - The statement is backward. Innovation strategies focus on long-term objectives, which sometimes do not work in the short run.

Isaac is looking for ways to offer new goods and services to his existing customers. He is pursuing a market development strategy

F - This is a product development strategy. A market development strategy offers existing products and services to new customers

Socially responsible marketing is defined as a strategic effort by firms to supply customers with environmentally friendly merchandise

F - This is the definition of green marketing.

When Sony released its PlayStation 3 game machines, it charged a high price, attracting the most avid game players. This was a market penetration pricing strategy

F - This was a price skimming strategy, which starts with a high price and then gradually lowers it.

The Value and Lifestyle Survey (VALS) conducted by Strategic Business Insights (SBI) is a widely used tool for geographic segmentation

F - VALS is widely used for psychographic segmentation

HappyCow is an example of a location-based social media application

True - Several restaurant chains are exploiting location-based social media applications, such as HappyCow VeginOut, Yelp, Foodspotting, inBloom, and Alfred. By using location-based applications on mobile phones, customers can find restaurants that cater to their specific dietary requirements.

34. Which of the following scenarios is not a reason a firm would eliminate an item within a product line? A. The firm decided to refocus marketing efforts elsewhere. B. The firm must respond to evolving markets. C. The product undermined its own brand. D. The product being eliminated is unprofitable. E. The firm has decided to capture new markets.

34. E - All of these are reasons why firms might decide to delete a product from a product line, except the desire to capture new markets

100. From a retailer's perspective, a(n) ________ is helpful in order to know exactly what will arrive in a particular shipment from a supplier. A. ASN B. EDI C.UPC D. VMI E. RFID

100. A - An advanced shipping notice (ASN) is an electronic document that the supplier sends the retailer in advance of a shipment to tell the retailer exactly what to expect in the shipment.

100. Which of the following provides the information needed to confirm insights and help managers pursue appropriate courses of action? A. quantitative research B. qualitative researc C. observation research D. in-depth interviews E. social media monitoring

100. A - Quantitative research aims to confirm insights and offer decision-making guidance to managers

105. The __________ is the primary enforcement agency for mass media advertising. A. FDA B. FCC C. FTC D. BATF E. USPS

105. C - The Federal Trade Commission (FTC) is the primary enforcement agency for most mass media advertising.

110. In a(n) _______ pricing strategy, marketers rely on the promotion of sales, during which prices are temporarily reduced to encourage purchases. A. High/low B. EDLP C. price skimming D. uniform delivered E. bait-and-switch

110. A - High/low pricing strategies adopt a regular, higher price and then occasionally offer lower sales prices to encourage purchases

116.When Barbara realized she didn't have all of the technical information she needed to answer the customer's questions, she made a call to her office. Who would be most likely to provide appropriate assistance? A. sales support personnel B. an order taker C. another salesperson D. her sales manager E. an order getter

116. A - Sales support personnel enhance and help with the overall selling effort, including providing technical information. A sales manager might be helpful, but may not know the technical information required

119. Both major political parties have developed proprietary databases that contain vast information about voters, broken down by demographic and geographic information. This kind of information is A. primary data. B. secondary data. C. syndicated data. D. neuromarketing. E. qualitative research.

119. B - In political settings, such secondary data can be critical for candidates running for office.

22. Kelly is the head of marketing for a nonprofit agency that supports the arts. She just received the go-ahead from her board of directors to conduct the agency's first-ever advertising campaign. Her first step will be to A. identify the target audience. B. set the advertising objectives. C. determine the advertising budget. D. evaluate and select the media. E. create the advertisements.

22. A - The first step in the development of an advertising campaign is to clearly identify the target market.

29.Thanh is starting a career in selling, but he eventually wants to become a senior manager. A selling career may help Thanh achieve that goal by providing A. structure. B. scheduling flexibility. C. job security. D. income. E. visibility to management.

29.E - Because of the salesperson's role as frontline emissary to customers, and because salespeople generate revenue, sales reps tend to be visible to management. This can be helpful to someone aiming for an executive's job.

40. In the communication process, the __________ is the medium that carries the message A. feedback loop B. sender C. transmitter D. communication channel E. receiver

40. D - The message is carried on a communication channel

When you register your telephone number with the National Do Not Call Registry, you are responding to a need for A. privacy. B. enhanced information flow. C. lower phone bills. D. marketing contact. E. tactical communication skills.

A - Privacy is a major concern for consumers, and the Federal Trade Commission is responding by trying to protect consumers against unwanted telephone solicitations.

Which of the following is the best way to build a sustainable competitive advantage using product excellence? A. being the first to offer customers desired features, even if competitors can copy them easily B. positioning the product using a clear, distinctive brand image C. having the most features on each model D. focusing on being cutting edge and continually eliminating older features that are still in use by customers E. copying the market leader's features, but at a lower cost

B - The incorrect options all describe ways to create new product models that are probably easy for competitors to copy (or that, in one case, copies competitors' offerings). A clear, distinct positioning, though, offers a more sustainable advantage through product excellence

59. When entering a foreign market, the least risky strategy is A. franchising. B. exporting. C. joint venture. D. direct investment. E. strategic alliance.

B - The least risky strategy is exporting because the firm does not own or establish anything in the country. The firm simply ships its products to buyers (whether consumers or business buyers) in the foreign country

The process of dividing the market into groups of customers with different needs, wants, or characteristics is called A. target marketing. B. market segmentation. C. positioning. D. allocation. E. value capture.

B - The process of dividing the market into groups of customers with different needs, wants, or characteristics—who therefore might appreciate products or services geared especially for them—is called market segmentation

Which of the following is the third step in the marketing planning process? A. define the business mission B. identify and evaluate opportunities C. evaluate using a matrix D. implement marketing mix and allocate resources E. perform situation analysis

B - The steps, in order, are: (1) define the business mission, (2) perform a situation analysis, (3) identify and evaluate opportunities, (4) implement marketing mix and allocate resources, and (5) evaluate performance

Most banks now have customer relationship software that, when a customer contacts the bank, tells the service representative what types of accounts, loans, and credit cards the customer currently has. Service representatives use this information to sell some of the other services the bank currently offers to these customers. This is a ___________________ growth strategy. A. product proliferation B. market development C. market penetration D. diversification E. product development

C - Selling more existing services to existing customers is a market penetration growth strategy. If these were newly developed services, this would be a product development example.

One of the benefits of a differentiated targeting strategy is that it allows the firm to diversify its business and A. decrease advertising spending. B. focus on a single market segment. C. lower overall risk. D. use undifferentiated strategies in micro markets. E. more widely market basic commodities.

C - Since a differentiated targeting strategy addresses multiple target markets, the firm can diversify its approach and potentially lower overall risk by not relying entirely on one target market. On the other hand, differentiated strategies can be expensive because of the need to communicate with multiple market segments.

Marketers wanting to use social media to reach consumers should understand that _______ were born into a world that was already full of electronic gadgets and digital technologies such as the Internet and social networks. A. Baby Boomers B. Generation W C. Generation X D. Generation Y E. Generation Z

E - Members of Generation Z are also known as Digital Natives since they have always had electronic gadgets and digital technologies in their world

Quick response (QR) systems allow retailers to maximize their inventory holdings

Quick response (QR; also known as just-in-time) systems are inventory management systems designed to minimize inventory levels (and thus inventory-carrying costs).

The greater the difference between a consumer's unsatisfied need and the desired state, the greater the need recognition will be.

T - A greater discrepancy leads to a more urgent drive to satisfy the need

Cheryl wants to quickly establish a dominant market share for her new line of ergonomic pens. To do this, she will likely use a market penetration pricing strategy

T - A market penetration pricing strategy starts with a low price in order to capture market share

How a product or service will be conceived or designed, how much it should cost, where and how it will be promoted, and how it will get to the consumer are all elements of A. a marketing plan. B. a marketing exchange. C. supply chain logistics. D. production management. E. delivery of the value proposition.

A - A marketing plan specifies the marketing activities for a specific period of time and is broken down into various components—how the product or service will be conceived or designed, how much it should cost, where and how it will be promoted, and how it will get to the consumer

Yolanda is the new restaurant manager in a major hotel. When considering changes in the restaurant that will increase value to customers, Yolanda will likely attempt to either provide the same quality at a lower cost or A. improve products and services at the same cost. B. increase prices to increase revenue. C. offset higher hotel rates with lower restaurant prices. D. reduce customer expectations through reduced service. E. lower the quality and the price.

A - By improving products at the same cost, Yolanda would create additional value for her customers.

If a radio station holds an online contest in which you must log in to its website and submit personal details such as name, phone number, and e-mail in order to participate, the radio station is A. offering an exchange. B. behaving unethically. C. hoping to receive feedback. D. implementing a CRM program. E. overstepping its role.

A - Marketing is about an exchange, the trade of things of value between buyer and seller so that each is better off. In this instance, the exchange is e-mail or personal information for a chance to win a contest. CRM would imply closer tracking of customers than this simple exchange suggests

Julia is considering a career in marketing. She is concerned about the image of marketers as fast-talking, high-pressure people who care only about making a sale. When reading about the core aspects of marketing, Julia is relieved to see that in marketing A. all parties to an exchange should be satisfied. B. promotion is the most important consideration, followed by pricing decisions. C. decisions are made regarding how a product is designed. D. customers are not considered until the product is ready for sale. E. distribution is controlled by customers.

A - Marketing is about satisfying customer needs and wants. If all parties to the exchange are satisfied, that indicates that marketers are considering customers' needs as well as the company's welfare

When conducting a SWOT analysis, in what phase of the strategic marketing process is an organization presently engaged? A. planning B. implementation C. control D. segmentation E. metrics

A - SWOT analysis is part of the planning phase

Frazier is out of milk and bread and needs to decide what is for dinner. He will be stopping at the grocery store on the way home. Frazier will likely engage in limited problem solving.

T - Milk and bread might be habitual purchases, or Frazier might consider different types of bread or different sized containers of milk. He must also decide on a menu for dinner and purchase the groceries he needs. None of this requires extended problem solving—in each case he will probably consider only a few alternatives—but a small amount of thought will be needed. This is an example of limited problem solving

Companies are legally required to disclose their privacy practices to customers on an annual basis

T - National and state governments in the United States play a big part in protecting privacy. Companies are legally required to disclose their privacy practices to customers on an annual basis.

The zone of tolerance refers to the area between customers' expectations regarding their desired service and the minimum level of acceptable service

T - Pertaining to the area between customers' expectations regarding their desired service and the minimum level of acceptable service, the zone of tolerance is the difference between what the customer really wants and what he or she will accept before going elsewhere

After purchasing an expensive pair of shoes, you may question whether the shoes are any better than the less expensive shoes you could have purchased instead. This is an example of postpurchase cognitive dissonance

T - Postpurchase cognitive dissonance is an internal conflict that arises from an inconsistency between two beliefs, or between beliefs and behavior. In this case, someone who spends a lot of money on shoes may feel a conflict due to the possibility that a cheaper pair would have satisfied the need for shoes just as well, leaving the extra money available for another use

Price is the only part of the marketing mix that does not generate costs

T - Product, place, and promotion all generate costs; price generates revenue

A delivery gap is the difference between the firm's service standards and the actual service it provides to customers

T - The Gaps Model allows systematic examination of all aspects of the service delivery process. The delivery gap represents the difference between the service standards and the actual service delivered

Successful firms focus their efforts on satisfying customer needs that match their core competencies

T - The firm's competencies are its strengths; the firm is likely to have the most success if it focuses on these, finding customers whose needs it can meet using these strengths

Members of Generation Y are also called Millennials

T - The group of people born between 1977 and 2000 are referred to as Generation Y, Gen Y, or Millennials.

Maintaining a unique value proposition can be sustained in the long term only in monopoly situations or monopolistic competition situations

T - The value proposition communicates the customer benefits to be received from a product or service and thereby provides reasons for wanting to purchase it. It is likely not sustainable, because competitors would attempt to copy the important product or service attributes and therefore begin to encroach on the firm's value proposition.

A delivery gap always results in a service failure

T - This is true; delivery gaps mean that service was not provided in an acceptable manner

The fact that the Apple iPad is easy to try—demo units are available at Apple stores as well as other retail stores—is helping it diffuse more quickly

T - This refers to trialability—the ability of a person to easily try a product. Trialability helps a product be adopted more quickly (i.e., to diffuse faster).

Lisle Hair Company keeps track of the gender and age of its customers so it can target e-mails to them and be sure to have the right hair products in stock when they visit. This is an example of the use of demographics in marketing

T - Typical demographics include age, gender, income, and race.

The idea that corporate social responsibility is unnecessary because the goal of any corporation is simply to make a profit has been thoroughly discredited and is no longer supported by economists or businesspeople

T - While some feel that the recent economic crisis argues in favor of social responsibility being a core responsibility of firms, not everyone agrees

A local art gallery keeps information on its customers regarding their preferences for certain artists as well as the style of art that interests them. The gallery uses this information to inform the customers when new products arrive from their favorite artists and targets them with special promotions. In this way, the gallery is using _______ to build loyalty among its customers. A. value cocreation B. customer relationship management C. transactional marketing D. B2B marketing E. the supply chain

B - Firms that employ customer relationship management (CRM) focus on identifying and building loyalty among the firm's most valued customers. They do so by systematically collecting information about their customers' needs and then use that information to target their best customers with the products, services, and special promotions that appear most important to them.

Yesenia, the new university course scheduling manager, is struggling with adjustments to the fall schedule. She is trying to determine how to offer the classes students need at the times when students need them. Yesenia is struggling with the marketing function of A. communicating the value proposition. B. managing the supply chain. C. creating value. D. capturing value. E. cocreating value.

B - Getting a product (in this case, a class) to customers when and where they want it describes the place component of the marketing mix, which involves managing the supply chain

The prevailing marketing strategy of the __________ era was to find customers for inventories that went unsold. A. production-oriented B. sales-oriented C. market-oriented D.value-based marketing E. retailing-oriented

B - In the sales-oriented era, manufacturers had the capacity to produce more than customers really wanted or were able to buy. Firms found an answer to their overproduction in becoming sales oriented; they depended on heavy doses of personal selling and advertising

Which of the following statements reflects the philosophy of the market-oriented era? A. A good product will sell itself. B. The customer is king. C. Firms should take advantage of a seller's market. D. Advertising and personal selling should be emphasized in order to make the sale. E. Firms should focus on value.

B - The market-oriented era was when most companies first started thinking in terms of meeting customer needs. It was a buyer's market and the customer was king

If a firm adopts a CRM business philosophy, it most likely has a(n) _______ orientation with its customers. A. transactional B. external C. relational D. internal E. divisional

C - A relational orientation refers to the building of relationships and the development of a better understanding of customers' needs and wants. A customer relationship management (CRM) program uses a set of strategies, programs, and systems that focus on identifying and building loyalty among the firm's most valued customers

Carla has been directed by her regional marketing manager to cut prices on seasonal items, place an ad in the local paper, and tell distributors to reduce deliveries for the next month. Which step of the strategic marketing planning process is Carla engaged in? A. evaluate performance B. define the business mission C. perform situation analysis D. implement marketing mix and resources E. identify and evaluate opportunities

D - Carla is dealing with implementation of the marketing mix—price cuts (price), advertising (promotion), and distribution changes (place).

Marketers want their firms to develop excellent supply chain management and strong supplier relations so they can A. persuade stores to refuse to carry competitors' products. B. use their power within the supply chain to force weaker firms to accept less favorable pricing. C. control prices and lock in margins. D. create a sustainable competitive advantage. E. justify charging higher prices than their competitors.

D - Some of the potential answers here represent unethical (and potentially illegal) behavior. The best answer is far simpler—this type of operational excellence helps create a sustainable competitive advantage by keeping costs low

Four Winds Art Gallery recently began offering appraisals of customers' art collections, in addition to continuing to sell paintings. Four Winds is A. expanding from offering just services to also offering goods. B. implementing a market segmentation strategy. C. capturing value through multiple pricing strategies. D. expanding from offering just goods to also offering services. E. increasing customer value through inflated appraisal evaluations.

D - The art gallery has been selling paintings, which are goods; offering appraisals are services

Which element of the marketing mix is most relevant to the activity "capturing value"? A. promotion B. purchasing C. product D. price E. place

D - The marketing mix activities are promotion (communicating value), product (creating value), price (capturing value), and place (delivering value).

Suppose your university made a sizable investment in its career services—additional counselors, increased efforts to bring in recruiters, and other services aimed at helping students find jobs. This investment would enhance the university's _____________ in an attempt to create value for students and recent graduates. A. segmentation strategy B. place strategy C. locational excellence strategy D. diversification strategy E. product value

E - The product the students buy—a degree—is enhanced by the ability to find a good job after graduation. Thus, this additional investment is related to the product value

According to your text, in the broadest terms, the "marketplace" refers to A. wholesale and retail environments. B. brick-and-mortar stores and the Internet. C. the four Ps. D. channels that are accessible to a given customer. E. the world of trade.

E - Using the broadest terms, the marketplace refers to the world of trade

Universal product codes (UPCs) are used to describe products for inventory taxation purposes

F - A UPC tag is the black-and-white bar code found on most merchandise. It contains a 13-digit code that indicates the manufacturer of the item, a description of the item, information about special packaging, and special promotions. It records receipt of the merchandise as it arrives at a distribution center

A survey of marketing officers reported that the most frequently observed type of unethical behavior was false or misleading advertising.

F - Although false/misleading ads were one category of often-observed unethical behavior, the most commonly reported behaviors were high-pressure, misleading, or deceptive sales tactics.

Applying age as a basis to identify customers is unethical, as it involves stereotyping

F - Applying age as a basis to identify consumers is quite useful to marketers, as long as it is used in conjunction with other consumer characteristics

Brainstorming in the ethical decision-making framework occurs immediately following the identification of issues.

F - Brainstorming occurs after the firm has identified the stakeholders and their issues and gathered all available data.

Burger King no longer uses SpongeBob SquarePants to promote burgers and fries because the character is no longer popular with young children

F - Burger King no longer uses SpongeBob because new advertising guidelines do not allow linking unhealthy foods with cartoon characters

The maturity phase of the product life cycle is characterized by greatly reduced competition.

F - Competition intensifies in the maturity phase, because the only way a firm can gain customers is typically to take them away from the competition.

Corporate social responsibility refers to the coordinated actions of government organizations to address the ethical, social, and environmental impacts of business operations

F - Corporate social responsibility refers to voluntary actions taken by a company to address the ethical, social, and environmental impacts of its business operations and the concerns of its stakeholders

Latosha was employee of the month at Jersey Mike's Subs, primarily because she provided excellent customer service by serving food quickly. This relates to the reliability service dimension.

F - Helping customers and providing prompt service relates to responsiveness

A company that sells only multigrain, low-calorie bread should use an undifferentiated targeting strategy.

F - If a company had a very specific product (such as multigrain, low-calorie bread) then it would probably use a concentrated targeting strategy.

Setting high customer expectations is a good strategy that will help to avoid customer dissatisfaction in the long run

F - If expectations are set too high, it can lead to dissatisfied customers because the high expectations are so difficult to live up to

The "implement marketing mix" step of the strategic marketing planning process is part of the control phase

F - In the implementation phase, marketing managers identify and evaluate different opportunities by engaging in a process known as segmentation, targeting, and positioning (STP). They then are responsible for implementing the marketing mix using the four Ps

One reason retailers are reluctant to use distribution centers is that retail store space is typically much less expensive than space at a distribution center, so it is more cost-effective to store merchandise and get it ready for sale at the retail store rather than at a distribution center

F - It is advantageous to use a distribution center since space at a distribution center is typically less expensive than a retail store.

Whereas objective measures seek to assess salespeople's behavior, subjective measures are quantitative.

F - Objective measures are quantitative; subjective measures seek to assess salespeople's behavior.

Online sales training is used to replace the one-on-one interaction of on-the-job training for advanced selling skills

F - Online sales training may never replace the one-on-one interaction of on-the-job training for advanced selling skills, but it is quite effective and efficient for many other aspects of the sales training task

Many services marketers use training and standardization to reduce service perishability

F - Perishability refers to the fact that services cannot be produced in advance and stored for use in the future. It is often addressed by offering discounts designed to shift demand to less busy periods

Kellogg's sells many different breakfast cereals, including Corn Flakes, Rice Krispies, Frosted Flakes, Raisin Bran, and Special K. This is an example of the breadth of the Kellogg's product mix

F - Product line depth (not breadth) equals the number of products within a product line, as in this question.

Sellers using an EDLP pricing strategy often communicate their strategy through the creative use of a reference price

F - Reference prices are used in conjunction with a high/low pricing strategy. The reference price allows buyers to compare the actual selling price of the product with the "sale" price and helps facilitate their evaluation process

STP refers to segmentation, testing, and promotion.

F - STP stands for segmentation, targeting, and positioning

In training service providers, service quality goals should be general to allow for the various needs of consumers

F - Service quality goals should be specific in order to consistently deliver service that meet customers' expectations.

When a firm has a particular profit goal as its overriding concern, it will use target return pricing to meet the profit objective

F - Target return pricing is used when firms want to produce a specific return on their investment; target profit pricing is implemented when a firm has a particular profit goal as its overriding objective.

When analyzing the immediate environment and the macroenvironment, marketers must be careful to keep the firm at the center of all analyses

F - The consumer, not the firm, should be kept at the center of all activity

Product penetration is one of the four major growth strategies

F - The four major growth strategies are market penetration, product development, market development, and diversification

A complex product that is best described in writing will be difficult to explain to consumers who are unable to read. For this reason, a segment in which many consumers could not read would be considered unattractive because the consumers would not be responsive.

F - The problem here is not responsiveness, but reachability. The consumers might respond very well to the offer if they could receive the message; however, their failure to read will make it difficult to reach them with positioning messages

As it relates to positioning, a self-values map displays the position of products or brands in the consumer's mind.

F - The question describes a perceptual map, not a self-values map

Ethos Water donates 2 percent of its profits to children in need of clean water. This action demonstrates that Ethos Water is a firm with a strong ethical climate

F - This is an example of corporate social responsibility, as it is a voluntary action designed to help the community at large. While ethical firms are more likely to be socially responsible, the two do not always go hand in hand.

Coercive power is based on getting a channel member to behave in a certain way because of a contractual agreement between the two firms

F - This is the definition of legitimate power. Coercive power is when a firm threatens to punish a channel member for not undertaking certain tasks

Pure competition occurs when there are many firms competing for customers in a given market, but their products are differentiated

F - This is the definition of monopolistic competition. With pure competition, a large number of sellers offer standardized products or commodities that consumers perceive as substitutable, such as grains, gold, meat, spices, or minerals

For market penetration pricing to work, the product or service must be perceived as breaking new ground in some way

F - This is true of price skimming, not market penetration pricing.

Warren is conducting a marketing research project. He is not sure what questions to ask or what types of consumers he should talk to. Therefore, Warren should begin by conducting quantitative research.

F - Warren should start with qualitative research to gain insights into the research questions before attempting quantitative research

In the United States, the federal government has enacted comprehensive privacy laws for the Internet.

F - While national and state governments in the United States play a part in protecting privacy, the U.S. government has failed to enact comprehensive privacy laws for the Internet. Several states, however, are starting to consider legislation

A marketing channel and a supply chain are virtually the same

T - A marketing channel is the set of institutions that transfer and move goods from the point of production to the point of consumption; this is essentially the same as a supply chain.

Relative market share is an example of a marketing metric.

T - A metric is a measuring system that quantifies a trend, dynamic, or characteristic. Relative market share might be used as a metric to evaluate a firm's performance compared to its competitors

Geraldo manages the electrical turbine engine division of General Electric Corporation. He makes most decisions independently, without consulting headquarters. Geraldo manages a strategic business unit.

T - A strategic business unit is a division of the firm that can be managed and operated somewhat independently from other divisions and may have a different mission or objectives

Stakeholders typically include the firm's employees, customer groups, stockholders, and members of the community in which the firm operates

T - All of these individuals and groups have a stake in how ethical issues are resolved

For products like pencils and paper clips, marketers should probably use an undifferentiated targeting strategy

T - An undifferentiated targeting strategy, also called mass marketing, is appropriate when everyone can be considered a potential user of a product, and when the potential benefits are similar for most consumers

In the marketing of services, empowerment means allowing employees to make decisions about how service is provided to customers

T - Empowerment is especially important when service is more individualized

When two competing retailers have a disagreement, it is an example of a horizontal channel conflict

T - Horizontal channel conflict occurs between members at the same level of marketing channel, such as two competing retailers.

When managers understand what their service providers are facing on a day-to-day basis by directly observing them and talking to customers, it is called "management by walking around."

T - Management by walking around involves knowing what their service providers are facing on a day-to-day basis and talking directly to the customers with whom those service providers interact.

The marketing profession has been singled out as the root cause of a host of ethical problems in business.

T - Marketing ethical lapses have led to problems such as unethical advertising and the promotion of shoddy products.

It was during the market-oriented era that firms first discovered "marketing." In what timeframe did this occur? A. around the turn of the 20th century B. shortly before the Great Depression C. just after World War II D. during the Roaring Twenties E. during the civil rights movement

C - After World War II, soldiers returned home, got new jobs, and started families. At the same time, manufacturers turned from focusing on the war effort toward making consumer products. Manufacturers and retailers thus began to focus on what consumers wanted and needed before they designed, made, or attempted to sell their products and services. It was during this period that firms discovered marketing

LeBron James, Alex Rodriguez, and other athletes are paid huge sums of money by companies for celebrity endorsements. If endorsements by these athletes create a clear understanding among consumers of the companies' products in comparison to competing products, they can help with the firm's __________ strategy. A. product excellence B. targeting C. positioning D. segmentation E. customer excellence

C - Positioning is the creation of a clear, distinct understanding of what the product does or represents compared to competitors. Celebrity endorsers are often used to convey this message.

As use of the Internet took off, car manufacturers were tempted to sell directly to consumers, but decided instead to continue to sell through their existing dealer networks. The car manufacturers considered switching from __________ to __________ marketing. A. B2C; B2B B. B2C; C2C C. B2B; B2C D. B2B; C2C E. C2C; B2C

C - The car manufacturers considered switching from B2B marketing (where they sell cars to dealers, who then sell them to consumers) to B2C marketing (where the car manufacturers sell directly to consumers).

In delivering value, marketing firms attempt to find the most desirable balance between A. the need for value and the perception of value. B. explicit versus implicit value. C. the need to provide benefits to customers and keep down costs. D. the desire to satisfy customers and the desire to satisfy employees. E. the need for product improvement and the need for advertising.

C - The challenge for all firms is to provide the value customers expect while keeping costs low enough to allow the firm to be profitable

During the __________ era, manufacturers and retailers began to focus on what consumers wanted and needed before they designed, made, or attempted to sell their products. A. production-oriented B. sales-oriented C. market-oriented D. value-based marketing E. retailing-oriented

C - The market-oriented era was when most companies first started thinking in terms of meeting customer needs

Many U.S. companies first discovered marketing during the __________ era. A. production-oriented B. sales-oriented C. market-oriented D. value-based marketing E. retailing-oriented

C - The market-oriented era was when most companies first started thinking in terms of meeting customer wants and needs.

Even when large discount retailers enter a market, a few small, local retailers survive and prosper. These small retailers have probably developed a(n) ________ that allows them to survive. A. advertising campaign B. plan to evaluate results C. sustainable competitive advantage D. set of performance metrics E. SWOT analysis

C - The other items listed could help the retailers develop or evaluate a plan to achieve a sustainable competitive advantage, but on their own they will not be enough. Small retailers usually cannot compete with larger competitors on price. But they might survive by offering services their customers value (and are willing to pay extra to get), by offering specialty products not carried by larger retailers, or by locating in places where the larger stores don't want to (or can't) locate


Conjuntos de estudio relacionados

HIS 101 - Western Civilization to 1689 - Final Exam

View Set

Chemistry - Mr Larry - Final Notes

View Set

Colorado Life - The Exam Simulator

View Set

Which of the following are the end products for cellular respiration?

View Set